ONLINE ORTHOPEDIC MCQS SPORT07

ONLINE ORTHOPEDIC MCQS SPORT 07

1.          A 22-year-old college baseball pitcher reports the recent onset of anterior and posterosuperior shoulder pain in his throwing shoulder.  Examination shows a 15-degree loss of internal rotation, tenderness over the coracoid, and a positive relocation test.  Radiographs are normal, and an MRI scan without contrast shows no definitive lesions.  A rehabilitation program is prescribed.  Which of the following regimens should be initially employed?

 

1-         Stretching the posterior capsule and pectoralis minor tendon

2-         Stretching the posterior capsule and strengthening the subscapularis

3-         Stretching the posterior capsule and using shoulder plyometrics 

4-         Stretching the anterior capsule and strengthening all components of the
rotator cuff

5-         Stretching the anterior capsule and improving pitching mechanics

 

PREFERRED RESPONSE: 1

 

DISCUSSION: Throwing athletes, particularly pitchers, have a high incidence of shoulder pain.  Recent evidence suggests that posteroinferior capsular tightness and scapular dyskinesis may play a substantial role in the pathologic cascade, culminating in the development of articular surface rotator cuff tears and tearing of the posterosuperior labrum.  These patients have posterosuperior shoulder pain primarily.  Furthermore, these athletes are susceptible to a muscular fatigue syndrome, the SICK (Scapular malposition, Inferior medial border prominence, Coracoid pain and malposition, and dysKinesis of scapular movement) scapula syndrome.  This patient has an internal rotation deficit and tenderness over the coracoid.  The internal rotation deficit is addressed by stretching the posterior capsule.  The tenderness over the coracoid has been attributed to a contracture of the pectoralis minor tendon secondary to scapular malposition.  The initial phase of the rehabilitation regimen is directed at stretching the posterior capsule and pectoralis minor tendon.

 

REFERENCES: Burkhart SS, Morgan CD, Kibler WB: The disabled throwing shoulder: Spectrum of pathology. Part III.  Arthroscopy 2003;19:641-661.

Kibler WB, McMullen J: Scapular dyskinesis and its relationship to shoulder pain.  J Am Acad Orthop Surg 2003;11:142-151.

 

2.         A 28-year-old professional football player reports painless loss of ankle motion after sustaining a “severe” ankle sprain 12 months ago.  A mortise radiograph is shown in Figure 1.  Surgical treatment should be reserved for which of the following conditions?

 

1-         Chronic ankle instability

2-         Persistent pain

3-         Progressive loss of ankle plantar flexion

4-         Development of ankle arthritis

5-         Instability of the proximal tibiofibular joint

 

PREFERRED RESPONSE: 2

 

DISCUSSION: The radiograph shows posttraumatic tibiofibular synostosis.  This condition typically follows an eversion (high) ankle sprain that results in disruption of the interosseous membrane.  Ossification usually develops within 6 to 12 months after the injury.  Return to sports is possible despite the lack of normal ankle dorsiflexion and mobility between the tibia and fibula.  Surgical excision is reserved for persistent pain that fails to respond to nonsurgical management once the ossification is “cold” on bone scintigraphy. 

 

REFERENCES: Whiteside LA, Reynolds FC, Ellsasser JC: Tibiofibular synostosis and recurrent ankle sprains in high performance athletes.  Am J Sports Med 1978;6:204-208.

Henry JH, Andersen AJ, Cothren CC: Tibiofibular synostosis in professional basketball players.  Am J Sports Med 1993;21:619-622.

Andrish J: The leg, in Drez D, DeLee JD, Miller MD (eds): Orthopaedic Sports Medicine Principles and Practice, ed 2.  Philadelphia, PA, WB Saunders, 2003, pp 2155-2181.

 

3.         The most common mechanism of injury to the triangular fibrocartilage complex
(TFCC) involves

 

1-         wrist extension and forearm pronation.

2-         wrist extension and forearm supination.

3-         wrist flexion and forearm pronation.

4-         wrist flexion and forearm supination.

5-         axial load in ulnar deviation.

 

PREFERRED RESPONSE: 1

 

DISCUSSION: TFCC tears are common in athletes.  As the athlete braces for a fall, the wrist is most commonly in an extended position and the forearm is pronated.

 

REFERENCES: Cohen MS: Ligamentous injuries of the wrist in the athlete.  Clin Sports Med 1998;17:533-552.

Rettig AC: Elbow, forearm and wrist injuries in the athlete.  Sports Med 1998;25:115-130.

 

4.         The force generated by a muscle is most highly dependent on its

 

1-         cross-sectional area.

2-         fiber type. 

3-         length.

4-         morphology.

5-         level of conditioning.

 

PREFERRED RESPONSE: 1

 

DISCUSSION: The cross-sectional area of a muscle determines to a great extent the force generated by the muscle.  The force of a muscle contraction is controlled by the amount of myofibrils that contract; the greater the amount of contracting myofibrils, the greater the force of contraction.  Fiber types have less to do with the force of contraction and more to do with the duration and speed of contraction.  Muscle length affects contraction force through the Blix curve.  The morphology of a muscle can affect the cross-sectional area by varying the angle of the fibers in relation to the force vector.  Conditioning mostly affects duration and fatigability.

 

REFERENCES: Buckwalter JA, Mow VC, Ratcliffe A: Restoration of injured or degenerated articular cartilage.  J Am Acad Orthop Surg 1994;2:192-201.

Buckwalter JA, Einhorn TA, Simon SR (eds): Orthopaedic Basic Science: Biology and Biomechanics of the Musculoskeletal System, ed 2.  Rosemont, IL, American Academy of Orthopaedic Surgeons, 2000, pp 683-716.

 

5.         A 31-year-old woman has increasing pain and tightness in her right knee, with occasional stiffness and recurrent hemorrhagic effusions.  MRI scans are shown in Figures 2a and 2b.  What is the most likely diagnosis?

 

1-         Rheumatoid arthritis

2-         Pigmented villonodular synovitis (PVNS)

3-         Synovial sarcoma

4-         Synovial chondromatosis

5-         Fibromatosis

 

PREFERRED RESPONSE: 2

 

DISCUSSION: PVNS is a rare inflammatory granulomatous condition of unknown etiology, and causes proliferation of the synovium of joints, tendon sheaths, or bursa. The disorder occurs most commonly in the third and fourth decades but can occur at any age.  MRI provides excellent delineation of the synovial disease.  Characteristic features of PVNS on MRI include the presence of intra-articular nodular masses of low signal intensity on T1- and T2-weighted images and proton density-weighted images.  Synovial biopsy should be performed if there is any doubt of the diagnosis.  Total synovectomy (open or arthroscopic) is required for the diffuse form, although recurrence is common.  Rheumatoid arthritis and synovial chondromatosis are not typically associated with hemorrhagic effusions.

 

REFERENCES: De Ponti A, Sansone V, Malchere M: Result of arthroscopic treatment of pigmented villonodular synovitis of the knee.  Arthroscopy 2003;19:602-607.

Chin KR, Barr SJ, Winalski C, et al: Treatment of advanced primary and recurrent diffuse pigmented villonodular synovitis of the knee.  J Bone Joint Surg Am 2002;84:2192-2202.

Bhimani MA, Wenz JF, Frassica FJ: Pigmented villonodular synovitis: Keys to early diagnosis. Clin Orthop 2001;386:197-202.

 

6.         A 30-year-old elite marathon runner reports chronic pain over the lateral aspect of the distal right leg and dysesthesia over the dorsum of the foot with active plantar flexion and inversion of the foot.  Examination reveals a tender soft-tissue fullness approximately
10 cm proximal to the lateral malleolus.  The pain is exacerbated by passive plantar flexion and inversion of the ankle.  There is also a positive Tinel’s sign over the site of maximal tenderness.  There is no motor weakness, and deep tendon reflexes are normal.  Radiographs and MRI of the leg are normal.  What is the next most appropriate step
in management?

 

1-         Biopsy of the soft-tissue mass

2-         Epidural corticosteroid injection into the lumbar spine

3-         Four-compartment fasciotomy of the leg

4-         Fascial release and neurolysis of the superficial peroneal nerve

5-         Closure of the fascial defect of the superficial peroneal nerve

 

PREFERRED RESPONSE: 4

 

DISCUSSION: The patient has entrapment of the superficial peroneal nerve against its fascial opening in the distal leg.  It is typically exacerbated by passive or active plantar flexion and inversion of the foot, which leads to traction of the nerve as it exits this opening.  Treatment involves release of the fascial opening to reduce this traction phenomenon.  Closure of the defect will only aggravate the condition and potentially result in an exertional compartment syndrome.  A four-compartment fasciotomy is only indicated for an established compartment syndrome of the leg. 

 

REFERENCES: Styf J: Diagnosis of exercise-induced pain in the anterior aspect of the lower leg.  Am J Sports Med 1988;16:165-169.

Sridhara CR, Izzo KL: Terminal sensory branches of the superficial peroneal nerve: An entrapment syndrome.  Arch Phys Med Rehabil 1985;66:789-791.

Styf J: Entrapment of the superficial peroneal nerve: Diagnosis and results of decompression. 

J Bone Joint Surg Br 1989;71:131-135.

 

7.         A 21-year-old soccer player reports pain and is unable to straighten his knee following an acute injury during a game.  He is unable to continue to play.  An MRI scan is shown in Figure 3.  What is the next most appropriate step in management?

 

1-         No weight bearing

2-         Cortisone injection

3-         Physical therapy

4-         Arthroscopic meniscectomy or repair

5-         Anterior cruciate ligament reconstruction

 

PREFERRED RESPONSE: 4

 

DISCUSSION: The patient has a locked knee that cannot be fully extended.  This is most likely the result of the mechanical block of a bucket-handle tear that has flipped into the notch.  Also, the pain may be so severe that the muscle spasm prevents the knee from straightening out.  When the patient is anesthetized, the muscle spasm relaxes and the meniscus can be reduced out of the notch.  Arthroscopy is the treatment of choice.  A meniscal repair is usually possible in large bucket-handle tears because the meniscus is torn in the red-red zone where most of the vascular supply is located.  If the handle portion is badly frayed or damaged, a partial meniscectomy should be performed.  The classic finding on MRI is a “double PCL sign.”  This is due to the flipped portion of the meniscus in the notch.

 

REFERENCES: Critchley IJ, Bracey DJ: The acutely locked knee: Is manipulation worthwhile?  Injury 1985;16:281-283.

Bansal P, Deehan DJ, Gregory RJ: Diagnosing the acutely locked knee.  Injury 2002;33:495-498.

 

8.         When performing an inside-out lateral meniscal repair, capsule exposure is provided by developing the

 

1-         iliotibial band and biceps tendon interval, then retracting the lateral head of the gastrocnemius anteriorly.

2-         iliotibial band and biceps tendon interval, then retracting the lateral head of the gastrocnemius posteriorly.

3-         iliotibial band and biceps tendon interval, then retracting the lateral collateral ligament posteriorly.

4-         iliotibial band and biceps tendon interval, then splitting the lateral head of the gastrocnemius.

5-         lateral head of the gastrocnemius and biceps tendon interval, then retracting the biceps tendon anteriorly.

 

PREFERRED RESPONSE: 2

 

DISCUSSION: Capsular exposure for an inside-out lateral meniscal repair is performed by developing the interval between the iliotibial band and biceps tendon.  Posterior retraction of the biceps tendon exposes the lateral head of the gastrocnemius.  Posterior retraction of the gastrocnemius provides access to the posterolateral capsule. 

 

REFERENCES: Miller DB Jr: Arthroscopic meniscus repair.  Am J Sports Med 1988;16:315-320.

Nawab A, Hester PW, Caborn DN: Arthroscopic meniscus repair, in Miller MD, Cole BJ (eds): Textbook of Arthroscopy.  Philadelphia, PA, WB Saunders, 2004, pp 517-537.

 

9.         A 50-year-old man reports left shoulder pain and weakness after undergoing a lymph node biopsy in his neck 2 years ago.  Examination reveals winging of the left scapula.  Electromyography shows denervation of the trapezius.  Surgical treatment for this condition involves

 

1-         pectoralis transfer to the medial border of the scapula.

2-         pectoralis transfer to the inferior border of the scapula.

3-         lateral transfer of the levator scapulae only.

4-         lateral transfer of the levator scapulae and rhomboid minor and major.

5-         latissimus dorsi transfer.

 

PREFERRED RESPONSE: 4

 

DISCUSSION: The muscle transfer procedure most commonly performed for trapezius paralysis is the Eden-Lange procedure.  Trapezius paralysis in this patient is secondary to iatrogenic injury to the spinal accessory nerve during lymph node biopsy.  In this procedure, the levator scapulae and rhomboid minor and major muscles are transferred laterally.  Pectoralis transfer to the inferior border of the scapula is used as a dynamic transfer for serratus anterior winging.

 

REFERENCES: Kuhn JE, Plancher KD, Hawkins RJ: Scapular winging.  J Am Acad Orthop Surg 1995;3:319-325.

Langenskiold A, Ryoppy S: Treatment of paralysis of the trapezius muscle by Eden-Lange operation.  Acta Orthop Scand 1973;44:383-388.

Romero J, Gerber C: Levator scapulae and rhomboid transfer for paralysis of trapezius: The Eden-Lange procedure.  J Bone Joint Surg Br 2003;85:1141-1145.

 

10.       A 15-year-old female field hockey player sustains a blow to the mouth from a hockey stick.  Three front teeth are knocked out and shown in Figure 4.  In addition to calling a dentist immediately, what is the next best step in management?

 

1-         Place the teeth in an ice water bath.

2-         Pour normal saline solution on the teeth and then place them in milk.

3-         Have the player gargle with mouthwash and place the teeth in water.

4-         Clean the teeth with a toothbrush and then reimplant them.

5-         Clean the teeth with a toothbrush and place them on ice.

 

PREFERRED RESPONSE: 2

 

DISCUSSION: Tooth avulsions can occur in contact or collision sports.  An avulsed tooth is a medical emergency.  The likelihood of survival of the tooth depends on the length of time that the tooth is out of the socket and the degree to which the periodontal ligament is damaged.  The tooth should be handled only by the crown end and not the root end.  It can be rinsed of debris with water or normal saline solution.  The tooth should not be brushed or cleaned otherwise.  During transport, the tooth must be kept moist.  An avulsed tooth can be transported in whole milk, saliva, sterile saline solution, or commercially available kits with physiologic buffer solutions.  The tooth and the athlete should be transported to the dentist for reinsertion as soon as possible and preferably within an hour.

 

REFERENCES: Krasner P: Management of sports-related tooth displacements and avulsions.  Dent Clin North Am 2000;44:111-135.

Sullivan JA, Anderson SJ (eds): Care of the Young Athlete.  Rosemont IL, American Academy of Orthopaedic Surgeons, Elk Grove Village, IL, American Academy of Pediatrics, 2000, p 190.

Galante A: Facial trauma, in Baker CL (ed): The Hughston Clinic Sports Medicine Book.  Baltimore, MD, Williams & Wilkins, 1995, p 121.

 

11.        Commotio cordis is best treated with

 

1-         immediate cardiac defibrillation.

2-         the chest thump maneuver.

3-         IV fluids and hydration.

4-         epinephrine.

5-         albuterol inhalers.

 

PREFERRED RESPONSE: 1

 

DISCUSSION: Commotio cordis is a rare but catastrophic condition that is caused by blunt chest trauma.  It results in cardiac fibrillation and is universally fatal unless immediate defibrillation is performed.  Although case reports of successful use of the chest thump maneuver exist, the best method of treatment is cardiac defibrillation.  IV fluids, epinephrine, and albuterol inhalers are used to treat dehydration, anaphylactic shock, and bronchospasm respectively, and are not effective in the treatment of commotio cordis.

 

REFERENCES: McCrory P: Commotio cordis.  Br J Sports Med 2002;36:236-237.

Boden BP, Tacchetti R, Mueller FO: Catastrophic injuries in high school and college baseball players.  Am J Sports Med 2004;32:1189-1196.

 

12.        Which of the following is considered an advantage of arthroscopic distal clavicle excision compared with open distal clavicle excision?

 

1-         Fewer complications

2-         Lower infection rate

3-         Evaluation of the glenohumeral joint

4-         Preservation of the inferior acromioclavicular ligament

5-         Decreased surgical time

 

PREFERRED RESPONSE: 3

 

DISCUSSION: Arthroscopic versus open distal clavicle excision has the advantage of allowing evaluation of the glenohumeral joint arthroscopically prior to moving into the subclavicular and subacromial space to perform the distal clavicle excision.  This can be of value in both confirming the diagnosis as well as avoiding diagnostic errors.  Berg and Ciullo showed that

in 20 patients who underwent open distal clavicle excision that resulted in failure, 15 of those patients had a superior labral anterior posterior (SLAP) lesion.  Of these 15 patients who had the lesion treated surgically, 9 went on to a good to excellent result after the surgery was performed arthroscopically.  Fewer complications, lower infection rate, and decreased surgical time have not been documented in the literature.  Arthroscopic technique sacrifices the inferior acromioclavicular ligament and preserves the superior acromioclavicular ligament.

 

REFERENCES: Berg EE, Ciullo JV: The SLAP lesion: A cause of failure after distal clavicle resection.  Arthroscopy 1997;13:85-89.

Lemos MJ, Tolo ET: Complications of the treatment of acromioclavicular and sternoclavicular joint injuries, including instability.  Clin Sports Med 2003;22:371-385.

 

13.       A 40-year-old woman reports the atraumatic onset of severe knee pain and swelling after undergoing an uncomplicated elective cholecystectomy 1 week ago.  She denies any history of diabetes mellitus or HIV but has had occasional episodes of mild knee pain and swelling that have always responded to nonsteroidal anti-inflammatory drugs.  Radiographs are shown in Figures 5a and 5b.  A knee aspiration yields a WBC count of 35,000/mm3.  The aspirate should also yield which of the following findings?

 

1-         Strongly negative needle-shaped crystals

2-         Weakly positive birefringent rhomboid-shaped crystals

3-         Gross blood

4-         Gram-positive cocci

5-         Gram-negative rods

 

PREFERRED RESPONSE: 2

 

DISCUSSION: The radiographs reveal chondrocalcinosis of the menisci.  This is caused by calcium pyrophosphate crystals, which are weakly positive birefringent rhomboid-shaped crystals.  Frequently, this condition is asymptomatic; however, routine abdominal surgery may cause precipitation of these crystals and pain.  Gout, which is caused by strongly negative birefringent needle-shaped sodium urate crystals, is not associated with chondrocalcinosis and is rare in younger women.  Gross blood is uncommon without trauma.  Infection is not likely in a healthy patient who underwent uncomplicated surgery.

 

REFERENCES: Fisseler-Eckhoff A, Muller KM: Arthroscopy and chondrocalcinosis.  Arthroscopy 1992;8:98-104.

Hough AJ Jr, Webber RJ: Pathology of the meniscus.  Clin Orthop 1990;252:32-40.

 

14.       What is the maximum acceptable amount of divergence of the interference screw in the femoral tunnel from the bone plug of a bone-patellar tendon-bone graft in anterior cruciate ligament (ACL) reconstruction before pull-out strength is statistically decreased?

 

1-         0 degrees

2-         10 degrees

3-         15 degrees

4-         30 degrees

5-         45 degrees

 

PREFERRED RESPONSE: 3

 

DISCUSSION: In the early 1990s, a transition was made from a two-incision ACL reconstruction to a single-incision ACL reconstruction, and there was concern over divergence of the femoral screws.  It was shown radiographically that approximately 5% of the time, divergence of the screw was greater than 15 degrees from the bone plug.  In a bovine model, there was significant loss of pull-out strength with an increase in divergence from 15 degrees to 30 degrees.  Therefore, attempts should be made to minimize divergence to 15 degrees or less.

 

REFERENCES: Lemos MJ, Jackson DW, Lee TO, et al: Assessment of initial fixation of endoscopic interference femoral screws with divergent and parallel placement.  Arthroscopy 1995;11:37-41. 

Lemos MJ, Albert J, Simon T, et al: Radiographic analysis of femoral interference screw placement during ACL reconstruction: Endoscopic versus open technique.  Arthroscopy 1993;9:154-158.

 

15.       A 21-year-old professional ballet dancer reports a painful popping sensation over her right hip joint.  Examination reveals that symptoms are reproduced with hip flexion and external rotation.  Which of the following studies will best confirm the diagnosis?

 

1-         Radiographs

2-         Bone scan

3-         CT

4-         Ultrasonography

5-         MRI

 

PREFERRED RESPONSE: 4

 

DISCUSSION: The patient has snapping hip syndrome of the internal type, which is more common in ballet dancers.  It is caused by the iliopsoas tendon gliding over the iliopectineal line or the femoral head.  The diagnosis usually can be made by the history and physical examination.  Snapping is reproduced by hip flexion and extension or flexion with external rotation and abduction.  Conventional and dynamic ultrasonography will confirm the snapping structure.  Radiographs occasionally show calcifications near the lesser trochanter.  MRI can be used to rule out other diagnoses that can simulate snapping hip.

 

REFERENCES: Gruen GS, Scioscia TN, Lowenstein JE: The surgical treatment of internal snapping hip.  Am J Sports Med 2002;30:607-613.

Garrick JG (ed): Orthopaedic Knowledge Update: Sports Medicine 3.  Rosemont, IL, American Academy of Orthopaedic Surgeons, 2004, pp 139-153.

 

16.        The posterior circumflex artery provides blood supply to what portion of the
proximal humerus?

 

1-         Almost the entire humeral head

2-         Lesser tuberosity

3-         Lesser tuberosity and posteroinferior humeral head

4-         Posterior portion of the greater tuberosity only

5-         Posterior portion of the greater tuberosity and a small portion of the posteroinferior humeral head

 

PREFERRED RESPONSE: 5

 

DISCUSSION: The posterior circumflex artery provides blood supply only to the posterior portion of the greater tuberosity and a small posteroinferior portion of the humeral head.  The humeral head is supplied primarily by the anterolateral ascending branch of the anterior circumflex artery; the terminal branch of this artery is termed the arcuate artery.

 

REFERENCES: Norris TR (ed): Orthopaedic Knowledge Update: Shoulder and Elbow 2.  Rosemont, IL, American Academy of Orthopaedic Surgeons, 2002, pp 267-274.

Gerber C, Schneeberger AG, Vinh TS: The arterial vascularization of the humeral head:

An anatomical study.  J Bone Joint Surg Am 1990;72:1486-1494.

 

17.        Use of prophylactic knee bracing in contact sports participants results in which of
the following?

 

1-         Decreased incidence of anterior cruciate ligament injuries

2-         Decreased incidence of posterior cruciate ligament injuries

3-         Decreased incidence of medial collateral ligament injuries

4-         Decreased incidence of meniscal tears

5-         Decreased incidence of ankle injuries

 

PREFERRED RESPONSE: 3

 

DISCUSSION: Several studies have looked at the effects of knee bracing, and it appears to be effective in prophylactically decreasing the incidence of medial collateral ligament sprains.  Najibi and Albright reported that although evidence is not conclusive, bracing appears to help decrease the incidence of medial collateral ligament injuries.  Albright and associates showed similar findings.  Prophylactic knee braces have been associated with an increased incidence of ankle injuries.

 

REFERENCES: Albright JP, Powell JW, Smith W, et al: Medial collateral ligament knee sprains in college football: Effectiveness of preventive braces.  Am J Sports Med 1994;22:12-18.

Najibi S, Albright JP: The use of knee braces: Part 1. Prophylactic knee braces in contact sports.  Am J Sports Med 2005;33:602-611.

 

18.       A 22-year-old college football player reports shortness of breath and dyspnea after a tackle.  Examination reveals tachypnea, tachycardia, the trachea is shifted to the right, and there are decreased breath sounds on the left lung fields.  The first line of treatment on the field should be

 

1-         placement of a chest tube.

2-         insertion of a large gauge needle into the second intercostal space.

3-         cardiopulmonary resuscitation.

4-         administration of adrenaline.

5-         immediate transfer to the emergency department.

 

PREFERRED RESPONSE: 2

 

DISCUSSION: The patient has a tension pneumothorax.  This is a life-threatening emergency where air is trapped between the pleura and the lung, which prevents expansion of the lung.  This causes hypoxia and cardiopulmonary compromise.  The first line of treatment is to place a needle into the second intercostal space in the midclavicular line.  The athlete should then be transported to the emergency department for chest tube placement.  The athlete cannot return to play, and resuscitation is not necessary because he has not gone into cardiopulmonary arrest.

 

REFERENCES: Amaral JF: Thoracoabdominal injuries in the athlete.  Clin Sports Med 1997;16:739-753.

Perron AD: Chest pain in athletes. Clin Sports Med 2003;22:37-50.

 

19.        Anabolic steroid use has which of the following effects on serum lipoprotein levels?

 

1-         Decrease in low-density lipoprotein only

2-         Decrease in high-density lipoprotein only

3-         Increase in high-density lipoprotein only

4-         Increase in low-density lipoprotein only

5-         Increase in both high- and low-density lipoproteins

 

PREFERRED RESPONSE: 2

 

DISCUSSION: The use of anabolic steroids causes a decrease in high-density lipoprotein levels but has no effect on low-density lipoprotein levels.  An abnormally low high-density lipoprotein level should alert the physician to the possibility of steroid use in an athlete. 

 

REFERENCES: Hartgens F, Rietjens G, Keizer HA, et al: Effects of androgenic-anabolic steroids on apolipoproteins and lipoprotein (a).  Br J Sports Med 2004;38:253-259.

Blue JG, Lombardo JA: Steroids and steroid-like compounds.  Clin Sports Med

1999;18:667-689.

 

20.       A 20-year-old professional female jockey who is wearing a helmet is thrown from
her horse.  What is the most likely location of her injury?

 

1-         Face

2-         Head

3-         Neck

4-         Lower back

5-         Leg

 

PREFERRED RESPONSE: 5

 

DISCUSSION: The incidence of injury associated with horseback rising is estimated to

be one per 350 riding hours to one per 1,000 riding hours.  Of these injuries, approximately

15% to 27% are severe enough to warrant hospital admission.  Significant and serious injuries in equestrian activities are associated with recreational riders and those not wearing a helmet.  Head and spine injuries are more common in recreational and nonhelmeted riders.  Extremity injuries are more common in professional and helmeted riders.  Professional riders are less likely to be admitted to the hospital than recreational riders, and are about half as likely to be disabled

at 6 months after injury as recreational riders.

 

REFERENCES: Lim J, Puttaswamy V, Gizzi M, et al: Pattern of equestrian injuries presenting to a Sydney teaching hospital.  ANZ J Surg 2003;73:567-571.

Petridou E, Kediloglou S, Belechri M, et al: The mosaic of equestrian-related injuries in Greece.  J Trauma 2004;56:643-647.

 

21.        A 62-year-old man with a long history of right shoulder pain and weakness is scheduled to undergo hemiarthroplasty.  Based on the radiographs shown in Figures 6a through 6c, what preoperative factor will most affect postoperative functional outcome? 

 

1-         Humeral head erosion

2-         Glenoid erosion

3-         Rotator cuff integrity

4-         Status of the coracoacromial ligament

5-         Acromioclavicular arthritis

 

PREFERRED RESPONSE: 3

 

DISCUSSION: The radiographs reveal osteoarthritis and proximal humeral head migration.  Integrity of the rotator cuff must be questioned based on these radiographic changes.  The status of the rotator cuff is the most influential factor affecting postoperative function in shoulder hemiarthroplasty.  The coracoacromial ligament provides a barrier to humeral head proximal migration in the face of a rotator cuff tear.  The radiographs do not indicate significant humeral head or glenoid erosion.  Acromioclavicular arthritis is often asymptomatic.

 

REFERENCES: Iannotti JP, Norris TR: Influence of preoperative factors on outcome of shoulder arthroplasty for glenohumeral osteoarthritis. J Bone Joint Surg Am 2003;85:251-258.

Hettrich CM, Weldon E III, Boorman RS, et al: Preoperative factors associated with improvements in shoulder function after humeral hemiarthroplasty.  J Bone Joint Surg Am 2004;86:1446-1451.

 

22.       Which of the following complications is more likely with an inside-out repair technique compared to an all-inside techniques for a medial meniscus tear?

 

1-         Failure

2-         Intra-articular synovitis

3-         Peroneal nerve injury

4-         Saphenous nerve injury

5-         Arthrofibrosis

 

PREFERRED RESPONSE: 4

 

DISCUSSION: All of the answers are possible complications of meniscal repair.  There are large volumes of literature evaluating the results of meniscal repair, both for the all-inside technique, as well as the inside-out technique.  Failure rates are similar.  Intra-articular synovitis occurs with absorbable sutures and absorbable implants.  Peroneal nerve injuries are more common with the lateral-sided repairs.  Saphenous nerve injuries are more common with medial-sided tears.  Because of the incision required and the technique of tying over soft tissue, the risk of a saphenous nerve injury is greater with an inside-out technique than with an all-inside technique.

 

REFERENCES: Farng E, Sherman O: Meniscal repair devices: A clinical and biomechanical literature review.  Arthroscopy 2004;20:273-286.

Jones HP, Lemos MJ, Wilk RM, et al: Two-year follow-up of meniscal repair using a bioabsorbable arrow.  Arthroscopy 2002;18:64-69.

 

23.       Figure 7 shows the CT scan of a 22-year-old professional baseball pitcher who has
had elbow pain for the past 6 months despite rest from throwing.  Management should consist of

 

1-         cast immobilization for 6 weeks.

2-         brief immobilization followed by rest for 6 weeks.

3-         internal fixation with a compression screw.

4-         internal fixation with a tension band wire.

5-         bone stimulation.

 

PREFERRED RESPONSE: 3

 

DISCUSSION: The CT scan shows a stress fracture of the olecranon.  This injury is the result of repetitive abutment of the olecranon into the olecranon fossa, traction from triceps activity during the deceleration phase of the throwing motion, and impaction of the medial olecranon onto the olecranon fossa from valgus forces.  Fractures may be either transverse or oblique in orientation.  Initial treatment consists of rest and temporary splinting.  Electrical bone stimulation may also be considered.  Open fixation with a large compression screw is recommended when nonsurgical management has failed to provide relief.

 

REFERENCES: Ahmad CS, ElAttrache NS: Valgus extension overload syndrome and stress injury of the olecranon.  Clin Sports Med 2004;23:665-676.

Garrick JG (ed): Orthopaedic Knowledge Update: Sports Medicine 3.  Rosemont, IL, American Academy of Orthopaedic Surgeons, 2004, pp 101-111.

 

24.       A 17-year-old football player is injured during a play and reports abdominal pain that is soon followed by nausea and vomiting.  What organ has most likely been injured?

 

1-         Liver

2-         Pancreas

3-         Spleen

4-         Kidney

5-         Large intestine

 

PREFERRED RESPONSE: 3

 

DISCUSSION: The spleen is the most common organ injured in the abdomen as the result of blunt trauma.  It is also the most common cause of death because of an abdominal injury. The liver is the second most commonly injured organ.  Injury to the other organs is rare.  The diagnosis can be made with CT.  Treatment ranges from observation to splenectomy, depending on the severity of injury.

 

REFERENCES: Green GA: Gastrointestinal disorders in the athlete.  Clin Sports Med 1992;11:453-470.

Kibler WB (ed): ACSM’s Handbook for Team Physician.  Philadelphia, PA,

Williams & Wilkins, 1996, p 151.

 

25.       A 15-year-old high school soccer player collides with an opponent and is unconscious when the trainer arrives on the field.  He is conscious within 15 seconds, breathing appropriately, and denies any headache, neck pain, or nausea.  It is his first head injury.  Provided that the athlete is free of symptoms, when should he be allowed to return to athletic activity?

 

1-         Immediately

2-         After 30 minutes

3-         After 24 hours

4-         After 4 weeks

5-         Next season

 

PREFERRED RESPONSE: 4

 

DISCUSSION: The loss of consciousness indicates a grade 2 concussion, which necessitates a

4-week period out of sport.  The last week prior to return must be symptom-free and the athlete should not have symptoms in practice.

 

REFERENCES: Cantu RC: Return to play guidelines after a head injury.  Clin Sports Med 1998;17:45-60.

Stevenson KL, Adelson PD: Pediatric sports-related head injuries, in Delee JC, Drez D (eds): Orthopaedic Sports Medicine, ed 2.  Philadelphia, PA, WB Saunders, 2003, vol 1, p 781.

 

26.       What type of exercise is used early in the rehabilitation process to safely stimulate
co-contraction of the scapular and rotator cuff muscles?

 

1-         Resistive active motion

2-         Facilitated active motion

3-         Plyometrics

4-         Open kinetic chain

5-         Closed kinetic chain

 

PREFERRED RESPONSE: 5

 

DISCUSSION: Closed kinetic chain exercises are used early in the rehabilitation process.  The distal segment is fixed, and an axial load is applied which provides glenohumeral compression and reduces the demand on the rotator cuff.  These exercises stimulate co-contractions of the scapular and rotator cuff muscles, load scapular stabilizers, and facilitate active motion.  Facilitated active motion exercises use proximal segment motion to stimulate and facilitate motion in the target tissue.  These exercises are often performed in diagonal movements.  Resistive active motion exercises are used later in the rehabilitation process.  These are typically open kinetic chain exercises that involve active glenohumeral motion with extrinsic loads such as weights or exercise tubing.  During the later stages of upper extremity rehabilitation, plyometrics are added.  These exercises help to prepare the athlete for return to sport.  When performed at slower speeds, these exercises emphasize stabilization and control.  As the speeds increase, muscles begin to work in the stretch-shortening sequence associated with sports participation. 

 

REFERENCES: Garrick JG (ed): Orthopaedic Knowledge Update: Sports Medicine 3.  Rosemont, IL, American Academy of Orthopaedic Surgeons, 2004, pp 129-136.

Kibler WB: Shoulder rehabilitation: Principles and practice.  Med Sci Sports Exerc

1998;30:S40-50.

 

27.       Which of the following cardiac conditions is considered an absolute contraindication to vigorous exercise?

 

1-         Hypertrophic cardiomyopathy (HCM)

2-         Sclerosis of the aortic valve without stenosis

3-         Mild mitral valve regurgitation

4-         Left ventricular hypertrophy (LVH)

5-         Functional murmurs

 

PREFERRED RESPONSE: 1

 

DISCUSSION: Hypertrophic cardiomyopathy (HCM) accounts for up to 50% of cases of

sudden death in young athletes.  HCM phenotype becomes evident by age 13 to 14 years.  Those at higher risk include individuals with cardiac symptoms, a family history of inherited cardiac disease, and those with a family history of premature sudden death.  Echocardiography is useful for detecting structural heart disease, including the cardiomyopathies and valvular abnormalities.  Trained adolescent athletes demonstrated greater absolute left ventricular wall thickness (LVWT) compared to controls.  HCM should be considered in any trained adolescent male athlete with a LVWT of more than 12 mm (female of more than 11 mm) and a nondilated ventricle.  Adolescent and adult athletes differ with respect to the range of LVWT measurements, as a manifestation of left ventricular hypertrophy (LVH).  Differentiating LVH (“athlete’s heart”) from HCM involves looking at additional echocardiographic features.  Sharma and associates reported that adolescents with HCM had a small or normal-sized left ventricle (less than 48 mm) chamber size, while those with LVH had a chamber size at the upper limits of normal (52 mm to 60 mm).

 

REFERENCES: Sharma S, Maron BJ, Whyte G, et al: Physiologic limits of left ventricular hypertrophy in elite junior athletes: Relevance to differential diagnosis of athlete’s heart and hypertrophic cardiomyopathy.  J Am College Cardiol 2002;40:1431-1436.

Maron BJ, Spirito P, Wesley Y, et al: Development and progression of left ventricular hypertrophy in children with hypertrophic cardiomyopathy.  N Engl J Med 1986;315:610-614.

Pelliccia A, Culasso F, Di Paolo FM, et al: Physiologic left ventricular cavity dilatation in elite athletes.  Ann Intern Med 1999;130:23-31.

 

28.       A 36-year-old professional baseball player reports the acute onset of severe right groin pain while attempting to avoid being hit by a baseball while at bat.  Examination reveals tenderness, soft-tissue swelling, and ecchymosis in the right groin extending over the medial thigh.  MRI scans are shown in Figures 8a and 8b.  Management should consist of

 

1-         open repair.

2-         immobilization.

3-         aspiration and steroid injection.

4-         rest, ice, and rehabilitation.

5-         tenotomy.

 

PREFERRED RESPONSE: 4

 

DISCUSSION: The MRI scans reveal a severe avulsion injury of the adductor muscle from the pubic ramus with muscle edema and hemorrhage.  Injury to the adductor muscle group, a “pulled groin,” is caused by forceful external rotation of an abducted leg.  Pain is immediate and severe in the groin region.  Tenderness is at the site of injury along the subcutaneous border of the pubic ramus.  Treatment is dictated by the severity of the symptoms but generally consists of rest, ice, and protected weight bearing, followed by a rehabilitation program that begins with gentle stretching and progresses to resistance exercise with a gradual return to sports.  Immobilization should be avoided because this promotes muscle tightness and scarring.  No data exist to suggest that open repair yields a better outcome than nonsurgical management.  Tenotomy has been performed in high-level athletes with chronic groin pain following injury. 

 

REFERENCES: Gilmore J: Groin pain in the soccer athlete: Fact, fiction, and treatment.  Clin Sports Med 1998;17:787-793.

Irshad K, Feldman LS, Lavoie C, et al: Operative management of “hockey groin syndrome”:

12 years of experience in National Hockey League players.  Surgery 2001;130:759-766.

 

29.       An 18-year-old football player is injured after making a tackle with his left shoulder.  He has decreased sensation over the lateral aspect of the left shoulder and radial aspect of the forearm.  Motor examination reveals weakness to shoulder abduction and external rotation as well as elbow flexion.  He has decreased reflexes of the biceps tendon on the left side but full, nontender range of motion of the cervical spine.  What anatomic site has been injured?

 

1-         Fourth cervical nerve root

2-         Upper trunk of the brachial plexus

3-         Middle trunk of the brachial plexus

4-         Lateral cord of the brachial plexus

5-         Axillary nerve

 

PREFERRED RESPONSE: 2

 

DISCUSSION: The athlete has symptoms referable to the axillary, musculocutaneous, and suprascapular nerves resulting from an injury to the upper trunk of the brachial plexus.  This portion of the plexus is formed by contributions of the fourth through sixth cervical nerve roots.  This area is often contused or stretched following a tackling maneuver that results in either depression of the shoulder from contact at Erb’s point or traction of the upper plexus from forced stretching of the neck to the contralateral side.

 

REFERENCES: Schenck CD: Anatomy of the innervation of the upper extremity, in Torg JS (ed): Athletic Injuries to the Head, Neck, and Face, ed 2.  St Louis, MO, Mosby-Year Book, 1991.

Hershman EB: Brachial plexus injuries.  Clin Sports Med 1990;9:311-329.

 

30.       Which of the following is considered the most common long-term effect on the spine of a professional race horse jockey?

 

1-         Thoracic spondylosis

2-         Lumbar spondylosis

3-         Cervical spondylosis

4-         Cervical and thoracic spondylosis

5-         Cervical and lumbar spondylosis

 

PREFERRED RESPONSE: 5

 

DISCUSSION: Horseback riding is a sport that directly affects the jockey’s spine.  Tsirikos and associates reported the results of a study of 32 jockeys.  They found that equestrian sports, especially professional horseback riding, apart from the increased risk of direct spinal injury caused by a fall from the horse, can lead to progressive spinal degeneration as a result of repetitive trauma and increased physical stress on the spine.  It is associated with spondylosis of the cervical spine and lumbar spine.

 

REFERENCES: Tsirikos A, Papagelopoulos PJ, Giannakopoulos PN, et al:  Degenerative spondyloarthropathy of the cervical and lumbar spine in jockeys.  Orthop 2001;24:561-564.

Hordegen KM: The spine and horseback riding.  Schweiz Med Wochenschr 1975;105:668-675.

 

31.       An 18-year-old lacrosse player is diagnosed with infectious mononucleosis.  What is the recommendation for return to play?

 

1-         Full participation once symptoms resolve

2-         Full participation once the splenomegaly resolves

3-         Full participation 4 weeks after the onset of symptoms regardless of the size
of the spleen

4-         Full participation 4 weeks after both the onset of illness and findings of a
normal-sized spleen

5-         No participation for 8 weeks

 

PREFERRED RESPONSE: 4

 

DISCUSSION: Infectious mononucleosis commonly affects adolescents and young adults.  It is a febrile illness accompanied by acute pharyngitis.  Splenomegaly may occur and predispose the athlete to splenic rupture.  Splenic rupture has been reported in nonathletes as well as in patients with normal-sized spleens.  Clinical evidence supports a return to all sports 4 weeks after the onset of symptoms provided that the spleen has returned to normal size.

 

REFERENCES: Auwaerter PG: Infectious mononucleosis: Return to play.  Clin Sports Med 2004;23:485-497.

Kinderknecht JJ: Infectious mononucleosis and the spleen.  Curr Sports Med Rep

2002;1:116-120.

 

32.       A 30-year-old patient reports chronic medial knee pain and swelling.  Figure 9a shows an articular cartilage lesion observed during arthroscopy.  The surgeon decides to treat the lesion with the microfracture technique seen in Figure 9b.  A biopsy of the repaired tissue 1 year after treatment is likely to show which of the following findings?

 

1-         Fibrous tissue

2-         Bone

3-         Articular cartilage

4-         Fibrocartilage

5-         Type II collagen

 

PREFERRED RESPONSE: 4

 

DISCUSSION: Microfracture is a marrow stimulation technique where stem cells from the underlying subchondral bone marrow can form at the base of the lesion.  The rationale for this technique is based on these cells differentiating into cells that will produce an articular cartilage repair.  Biopsy findings in animals and humans have demonstrated primarily a fibrocartilagenous repair tissue and not articular cartilage.  The collagen type found in hyaline or articular cartilage is of the type II variety.  Fibrocartilage possesses mostly type I and III cartilage.

 

REFERENCES: Buckwalter JA, Mankin HJ: Articular cartilage: Degeneration and osteoarthritis, repair, regeneration, and transplantation.  Instr Course Lect 1998;47:487-504.

Buckwalter JA, Einhorn TA, Simon SR (eds): Orthopaedic Basic Science: Biology and Biomechanics of the Musculoskeletal System, ed 2.  Rosemont, IL, American Academy of Orthopaedic Surgeons, 2000, pp 471-488.

 

33.       A 24-year-old dancer reports posterior ankle pain when in the “en pointe” position.  Examination reveals posteromedial tenderness, no pain reproduction with passive forced planter flexion, and pain with motion of the hallux.  What is the most likely diagnosis?

 

1-         Painful os trigonum

2-         Posterior ankle soft-tissue impingement

3-         Stricture in the knot of Henry

4-         Flexor digitorum longus tendinitis

5-         Flexor hallucis longus tendinitis

 

PREFERRED RESPONSE: 5

 

DISCUSSION: Flexor hallucis longus tendinitis is a common cause of posterior ankle pain in dancers.  It tends to be more posteromedial and is characterized by a clicking or catching sensation posteromedially with motion of the great toe.  A painful os trigonum typically causes more posterolateral ankle pain and may occur after an ankle sprain or plantar flexion injury where there may be a fracture of the os trigonum.

 

REFERENCES: Garrick JG (ed): Orthopaedic Knowledge Update: Sports Medicine 3.  Rosemont, IL, American Academy of Orthopaedic Surgeons, 2004, pp 249-261.

Hamilton WG, Geppert MJ, Thompson FM: Pain in the posterior aspect of the ankle in dancers: Differential diagnosis and operative treatment.  J Bone Joint Surg Am 1996;78:1491-1500.

 

34.       Kinematic analysis of the medial and lateral menisci has demonstrated that the lateral meniscus has which of the following characteristics compared with the medial meniscus?

 

1-         More soft-tissue attachments/restraints

2-         More mobility

3-         Less mobility

4-         No posterior movement with flexion

5-         No anterior movement with extension

 

PREFERRED RESPONSE: 2

 

DISCUSSION: Kinematic analysis of both menisci demonstrates anterior movement with extension and posterior movement with flexion.  The lateral meniscus has more mobility than the medial meniscus because of less soft-tissue attachments.

 

REFERENCES: Insall JN, Scott WN (eds): Surgery of the Knee, ed 3.  New York, NY, Churchill Livingstone, 2001, vol 1, p 474.

Thompson WO, Thaete FL, Fu FH, et al: Tibial meniscal dynamics using 3D reconstructions

of MR images, in Proceedings of the Orthopaedic Research Society 1990;389.

Vaccaro AR (ed): Orthopaedic Knowledge Update 8.  Rosemont, IL, American Academy

of Orthopaedic Surgeons, 2005, pp 15-28.

 

35.       A 24-year-old professional basketball player reports the gradual onset of pain that is poorly localized to the left midfoot for the past 2 months.  Examination reveals diffuse tenderness to palpation, full range of motion of the ankle and subtalar joint, and a normal neurovascular examination to the foot.  An AP radiograph is shown in Figure 10.  Definitive treatment should include

 

1-         a custom-molded orthotic and anti-inflammatory drugs.

2-         partial weight-bearing ambulation with crutches.

3-         weight bearing as tolerated with a walking boot.

4-         casting for 6 weeks with bone stimulation.

5-         internal fixation.

 

PREFERRED RESPONSE: 5

 

DISCUSSION: The imaging studies reveal a navicular stress fracture.  This condition is secondary to chronic overuse (often running on hard surfaces) and results in vague, ill-defined pain in the midfoot.  These fractures can be missed on radiographs but are well-defined on CT or MRI.  Tarsal navicular fractures are typically oriented in the sagittal plane.  Surgery is typically indicated for the high-level athlete because of the high risk for nonunion and persistent symptoms following nonsurgical management.  Internal fixation is the treatment of choice.

 

REFERENCES: Torg JS, Pavlov H, Cooley JH, et al: Stress fractures of the tarsal navicular. 

J Bone Joint Surg Am 1982;64:700-712.

Brodsky JW, Krause JO: Stress fractures of the foot and ankle, in Drez D, DeLee JD, Miller MD (eds): Orthopaedic Sports Medicine Principles and Practice, ed 2.  Philadelphia, PA, WB Saunders, 2003, pp 2391-2409.

 

36.       A 19-year-old college football player reports persistent weakness, tingling, and numbness of both upper extremities at half time.  He states that these symptoms initially occurred after tackling an opposing player with his head early in the game.  History reveals that he has had “burners” in the past that typically resolved within 15 to 30 minutes.  Examination reveals pain-free cervical motion, weakness to shoulder abduction testing bilaterally, normal upper extremity reflexes, and decreased sensation over both shoulders and the upper arms.  Appropriate initial management should consist of

 

1-         no treatment, the athlete may return to play.

2-         modification of the shoulder pads and a return to play.

3-         shoulder rehabilitation exercises and a return to play when strength is normal.

4-         MRI of the cervical spine.

5-         CT of the brain.

 

PREFERRED RESPONSE: 4

 

DISCUSSION: The player’s symptoms represent more than the mere “burner syndrome,” which leads to unilateral symptoms that typically last less than 1 minute.  Return to play following a burner is allowed following nonsurgical management and once the symptoms have subsided and the player exhibits normal strength and motion of the neck and upper extremities.  This player has the history, symptoms, and examination findings that are consistent with cervical neurapraxia.  Return to play in contact sports is contraindicated with bilateral symptoms prior to MRI evaluation of the cervical spine.  CT of the brain is indicated with a history of loss of consciousness or other symptoms suggestive of a concussion.

 

REFERENCES: Torg JS, Sennett B, Pavlov H, et al: Spear tackler’s spine: An entity precluding participation in tackle football and collision activities that expose the cervical spine to axial energy inputs.  Am J Sports Med 1993;21:640-649.

Torg JS: Cervical spinal stenosis with cord neurapraxia and transient quadriplegia.  Sports Med 1995;20:429-434.

Torg JS, Guille JT, Jaffe S: Injuries to the cervical spine in American football players.  J Bone Joint Surg Am 2002;84:112-122.

 

37.       Which of the following is the most relevant clinical factor in the maturation
assessment of an adolescent female athlete contemplating anterior cruciate ligament (ACL) reconstruction?

 

1-         Parental height

2-         Height of older male sibling

3-         Age of menarche

4-         Recent change in shoe size

5-         Presence of breast buds

 

PREFERRED RESPONSE: 3

 

DISCUSSION: Age of menarche is the most accurate clinical factor to assess the degree of skeletal maturity in the female athlete. Such an assessment is necessary prior to ACL reconstruction in a skeletally immature female because of the risk of damage to the distal femoral and proximal tibial physes. Height of an older male sibling is not relevant to the female athlete.  Parental height and recent change in shoe size are only moderately useful in predicting final growth, and hence, skeletal maturity.  The presence of breast buds occurs early in adolescent development; therefore, its presence suggests a high likelihood of future growth.

 

REFERENCES: Micheli LJ, Foster TE: Acute knee injuries in the immature athlete.  Instr Course Lect 1993;42:473-481.

Stanitski CL: Anterior cruciate ligament injury in the skeletally immature patient: Diagnosis and treatment.  J Am Acad Orthop Surg 1995;3:146-158.

Fowler PJ: Anterior cruciate ligament injuries in the child, in Drez D, DeLee JD, Miller MD (eds): Orthopaedic Sports Medicine Principles and Practice, ed 2.  Philadelphia, PA, WB Saunders, 2003, pp 2067-2074.

 

38.       A 28-year-old woman fell on her right wrist while rollerblading 2 days ago.  She was seen in the emergency department at the time of injury and was told she had a sprain.  Examination now reveals dorsal tenderness in the proximal wrist but no snuffbox or ulnar tenderness.  Standard wrist radiographs are normal.  What is the next most appropriate step in management?

 

1-         Arthroscopy of the wrist

2-         CT of the wrist

3-         PA clenched fist radiograph

4-         Electromyography and nerve conduction velocity studies

5-         AP and lateral radiographs of the forearm

 

PREFERRED RESPONSE: 3

 

DISCUSSION: When considering the diagnosis of scapholunate ligament injury, standard radiographic views of the hand will not always reveal widening of the scapholunate gap.  Although MRI may reveal injury to the ligaments, the PA clenched fist view can be obtained in the office during the initial patient visit.  Arthroscopy is not a first-line diagnostic tool.

 

REFERENCES: Walsh JJ, Berger RA, Cooney WP: Current status of scapholunate interosseous ligament injuries.  J Am Acad Orthop Surg 2002;10:32-42.

Browner BD, Levine AM, Jupiter JB, et al (eds): Skeletal Trauma, ed 2.  Philadelphia, PA, WB Saunders, 1998, pp 1366-1367.

 

39.       A 17-year-old basketball player and pole vaulter who has had anterior knee pain for the past 18 months now reports a recent inability to jump.  Based on the MRI scan shown in Figure 11, management should consist of

 

1-         debridement and repair.

2-         cast immobilization.

3-         aggressive overload eccentric strengthening.

4-         ice massage and continued athletic participation.

5-         steroid injection.

 

PREFERRED RESPONSE: 1

 

DISCUSSION: The MRI scan reveals a partial patellar tendon rupture in conjunction with chronic patellar tendinitis.  Mild and moderate patellar tendinitis may be treated nonsurgically with rest, stretching, strengthening, and anti-inflammatory drugs.  Severe tendinopathy or extensor mechanism disruption is best treated surgically with tendon debridement and repair.

 

REFERENCES: Al-Duri ZA, Aichroth PM: Surgical aspects of patella tendonitis: Techniques and results.  Am J Knee Surg 2001;14:43-50.

Garrick JG (ed): Orthopaedic Knowledge Update: Sports Medicine 3.  Rosemont, IL, American Academy of Orthopaedic Surgeons, 2004, pp 218-219.

 

40.       Figures 12a through 12c show the radiographs of a 28-year-old professional baseball player who has ulnar-sided wrist pain and numbness and tingling in the fourth and fifth digits for the past 6 weeks.  Management should consist of

 

1-         cast immobilization.

2-         bone stimulation and splinting.

3-         ulnar nerve exploration.

4-         open reduction and internal fixation.

5-         excision of the fragment.

 

PREFERRED RESPONSE: 5

 

DISCUSSION: Hook of the hamate fractures typically occur as a result of direct force from swinging a bat, golf club, or racket.  Pain is localized to the hypothenar eminence.  The injury is best seen on a carpal tunnel view.  CT will confirm the diagnosis.  Chronic cases can be associated with neuropathy of the ulnar nerve.  Excision of the hook through the fracture site usually yields satisfactory results, allowing the athlete to return to competition.

 

REFERENCES: Parker RD, Berkowitz MS, Brahms MA, et al: Hook of the hamate fractures in athletes.  Am J Sports Med 1986;14:517-523.

Garrick JG (ed): Orthopaedic Knowledge Update: Sports Medicine 3.  Rosemont, IL, American Academy of Orthopaedic Surgeons, 2004, pp 113-128.

 

41.       Figure 13 shows the radiographs of a 20-year-old intercollegiate basketball player
who was injured 6 weeks prior to the start of the season.  What is the most
appropriate treatment?

 

1-         Intramedullary screw fixation

2-         Immobilizing orthotic/boot

3-         Walking cast

4-         Physical therapy

5-         Rest for 2 weeks and return to play

 

PREFERRED RESPONSE: 1

 

DISCUSSION: A Jones fracture occurs at the metaphyseal-diaphyseal junction of the fifth metatarsal.  It is often an acute fracture in conjunction with a chronic stress-related injury.  It requires either a short leg cast with strict non-weight-bearing or surgical fixation.  In the high performance athlete, the need for rapid return to sport activity usually requires surgical intervention, most commonly with an intramedullary screw.

 

REFERENCES: Brodsky JW, Krause JO: Stress fractures of the foot and ankle, in Delee JC, Drez D (eds): Orthopaedic Sports Medicine, ed 2.  Philadelphia, PA, Saunders, 2003, vol 2,

pp 2391-2409.

Dameron TB Jr: Fractures of the proximal fifth metatarsal: Selecting the best treatment option. 

J Am Acad Orthop Surg 1995;3:110-114.

 

42.       A 12-year-old boy reports the acute onset of pain and a pop over the right side of his pelvis while swinging a baseball bat during a Little League game.  Radiographs reveal an avulsion of the anterior superior iliac spine with 2 cm of displacement.  Management should consist of

 

1-         open reduction and internal fixation of the fragment along with the rectus femoris.

2-         open reduction and internal fixation of the fragment along with the sartorius.

3-         open reduction and internal fixation of the fragment along with the iliopsoas.

4-         rest and protected weight bearing with crutches.

5-         excision of the fragment.

 

PREFERRED RESPONSE: 4

 

DISCUSSION: Anterior superior iliac spine avulsion fractures are caused by sudden, forceful contractions of the sartorius and tensor fascia lata.  These injuries occur in young athletes through the growth plate with the hip extended and the knee flexed, such as while sprinting or swinging a baseball bat.  The athlete will often report a pop or snap at the time of injury.  Displaced fractures usually can be seen on radiographs.  CT or MRI can be obtained to confirm the diagnosis.  In most patients, nonsurgical management consisting of rest and protected weight bearing yields satisfactory outcomes.  Surgery is usually reserved for fractures with displacement of more than 3 cm and painful nonunions.

 

REFERENCES: Garrick JG (ed): Orthopaedic Knowledge Update: Sports Medicine 3. Rosemont, IL, American Academy of Orthopaedic Surgeons, 2004, pp 139-153.

White KK, Williams SK, Mubarack SJ: Definition of two types of anterior superior iliac spine avulsion fractures. J Pediatr Orthop 2002;22:578-582.

 

43.       Which of the following best describes heat stroke?

 

1-         Transient loss of consciousness with peripheral vasodilation and decreased cardiac output with normal body temperature

2-         A condition involving painful contractions of large muscle groups because of decreased hydration and a decrease of serum sodium and chloride

3-         Hypernatremia in poorly conditioned athletes, manifested by thirst and oliguria with a core temperature of less than 102.2 degrees F (39 degrees C)

4-         Hyperthermia, central nervous system dysfunction, and loss of thermoregulatory function

5-         A transient condition that responds to glucose administration

 

PREFERRED RESPONSE: 4

 

DISCUSSION: Heat stroke consists of hyperthermia (greater than 105.8 degrees F

[41 degrees C]), central nervous system dysfunction, and cessation of sweating with hot, dry skin.  It is a medical emergency that results from failure of the thermoregulatory mechanisms of the body.  It has a high death rate and requires rapid reduction in body core temperature.  Heat syncope is characterized by a transient loss of consciousness with peripheral vasodilation and decreased cardiac output with normal body temperature.  Heat cramps involve painful contractions of large muscle groups because of decreased hydration and a decrease of serum sodium and chloride.  Heat exhaustion is distinguished by a core temperature of less than

102.2 degrees F (39 degrees C) and an absence of central nervous system dysfunction.  Hypernatremic heat exhaustion results from inadequate water replacement.

 

REFERENCES: Knochel JP: Environmental heat illness: An eclectic review.  Arch Intern Med 1974;133:841-864.

Hubbard RW, Gaffin SL, Squire DL: Heat related illness, in Wilderness Medicine, ed 3. 

St Louis, MO, Mosby, 1995, p 167.

Khosla R, Guntupalli KK: Heat-related illnesses.  Crit Care Clin 1999;15:251-263.

 

44.       Which of the following factors is most critical to the success of a meniscal
allograft transplantation? 

 

1-         Accurate graft size

2-         Donor cell viability

3-         Reestablishment of the central meniscal blood supply

4-         Suppression of the immune response

5-         Cryopreservation of the donor graft

 

PREFERRED RESPONSE: 1

 

DISCUSSION: Success of a meniscal allograft transplantation is strongly dependent on accurate graft sizing, typically within 5% of the native meniscus.  Previous studies have established that donor cell viability is not mandatory for the survival of these grafts since they are replaced by the recipient’s cells (at least peripherally) within several weeks.  Thus, cryopreservation of the graft to ensure cell viability is not necessary.  There is a limited immune response to musculoskeletal allografts; therefore, immunosuppression, as is required for visceral organ transplantation, is not indicated.

 

REFERENCES: Wirth CA, Kohn D: Meniscal transplantation and replacement, in Fu FH, Harner CD, Vince JG (eds): Knee Surgery.  Baltimore, MD, Williams & Wilkins, 1994, vol 1,

pp 631-641.

Brautigan BE, Johnson DL, Caborn DM, et al: Allograft tissues, in Drez D, DeLee JD, Miller MD (eds): Orthopaedic Sports Medicine: Principles and Practice, ed 2.  Philadelphia, PA,

WB Saunders, 2003, pp 205-213.

Shaffer B, Kennedy S, Klimkiewicz J, et al: Preoperative sizing of meniscal allografts in meniscus transplantation.  Am J Sports Med 2000;28:524-533.

 

45.       What is the most common behavioral effect of anabolic steroid use in athletes?

 

1-         Increased aggression

2-         Psychosis

3-         Drug dependence

4-         Depression

5-         Mania

 

PREFERRED RESPONSE: 1

 

DISCUSSION: Users of anabolic steroids often display increased feelings of hostility and aggression.  Although reports of psychotic, depressive, and manic behavior have been reported with the use of steroids, they are rare.  Drug dependence, such as seen with narcotics, is not a feature of steroid use.

 

REFERENCES: Hartgens F, Kuipers H: Effects of androgenic-anabolic steroids in athletes.  Sports Med 2004;34:513-554.

Blue JG, Lombardo JA: Steroids and steroid-like compounds.  Clin Sports Med

1999;19:667-689.

 

46.       What is the effect on knee kinematics following placement of an anterior cruciate ligament (ACL) graft at the 12 o’clock position?

 

1-         Decreased rotational stability

2-         Decreased anterior-posterior stability

3-         Decreased flexion

4-         Decreased extension

5-         Graft failure secondary to impingement

 

PREFERRED RESPONSE: 1

 

DISCUSSION: Endoscopic ACL reconstructive techniques may result in a vertical graft placement.  The reconstructed ligament will resist anterior translation of the tibia but the graft will not restore rotatory stability.  Decreased flexion and extension are caused by placement of the femoral tunnel too anterior and posterior, respectively.  Impingement of the graft on the femoral notch is caused by anterior placement of the tibial tunnel or inadequate notchplasty.

 

REFERENCES: Scopp JM, Jasper LE, Belkoff SM, et al: The effect of oblique femoral tunnel placement on rotational constraint of the knee reconstructed using patellar tendon autografts.  Arthroscopy 2004;20:294-299.

Carson EW, Simonian PT, Wickiewicz TL, et al: Revision anterior cruciate ligament reconstruction.  Instr Course Lect 1998;47:361-368.

 

47.       The superior glenohumeral ligament primarily restrains

 

1-         posterior translation of the humeral head with the arm in 90 degrees of abduction.

2-         inferior translation of the humeral head with the arm in adduction.

3-         anterior translation of the humeral head in 90 degrees of abduction.

4-         anterior translation of the humeral head with the arm in 45 degrees of abduction.

5-         anterior translation of the humeral head with the arm in 90 degrees of abduction and external rotation.

 

PREFERRED RESPONSE: 2

 

DISCUSSION: Several cutting studies have evaluated the primary static restraints and the role of the glenohumeral ligaments in providing static stability.  With the arm at the side in adduction, the superior glenohumeral ligament and coracohumeral ligament are the primary restraints to inferior translation.  The middle glenohumeral ligament functions with the arm in 45 degrees of abduction and resists anterior translation.  The inferior glenohumeral ligament is the primary restraint to anterior translation at 90 degrees of abduction.

 

REFERENCES: Warner JJ, Deng XH, Warren RF, et al: Static capsuloligamentous restraints to superior-inferior translation of the glenohumeral joint.  Am J Sports Med 1992;20:675-685.

Griffin LY (ed): Orthopaedic Knowledge Update: Sports Medicine.  Rosemont, IL, American Academy Orthopaedic Surgeons, 1994, pp 165-177.

 

48.       Which of the following best describes carbohydrate loading?

 

1-         A method of improving the performance of nonendurance athletes

2-         A technique to improve athletic performance without any deleterious side effects

3-         Tapering caloric intake along with training the week before an endurance event

4-         The practice of decreasing training and increasing carbohydrate intake the week before an endurance event

5-         Dietary manipulation to improve glycogen stores by increasing athletic activity and decreasing carbohydrate intake

 

PREFERRED RESPONSE: 4

 

DISCUSSION: Carbohydrate loading is the practice of maximizing glycogen stores by decreasing training and increasing carbohydrate intake the week before an endurance event.  Nonendurance athletes do not benefit from this because glycogen depletion is not the limiting factor during a normal competition.  Potential side effects of carbohydrate loading are water retention, muscle stiffness, and weight gain. 

 

REFERENCES: Coyle EF, Hagberg JM, Hurley BF, et al: Carbohydrate feeding during prolonged strenuous exercise can delay fatigue.  J Appl Physiol 1983;55:230-235.

Costill DL, Sherman WM, Fink WJ, et al: The role of dietary carbohydrates in muscle glycogen resynthesis after strenuous running.  Am J Clin Nutr 1981;34:1831-1836.

Sherman WM, Doyle JA, Lamb DR, et al: Dietary carbohydrate, muscle glycogen, and exercise performance during 7 d of training.  Am J Clin Nutr 1993;57:27-31.

 

49.       A 29-year-old quarterback falls onto his dominant shoulder and sustains the injury shown in Figures 14a and 14b.  Management should consist of

 

1-         an arm sling.

2-         nonsteroidal anti-inflammatory drugs and a rapid return to activity.

3-         arthroscopic partial claviculectomy.

4-         acromioclavicular joint reduction and stabilization.

5-         acromionectomy.

 

PREFERRED RESPONSE: 4

 

DISCUSSION: Type V acromioclavicular dislocations are characterized by elevation of the clavicle of 100% to 300% and involve extensive soft-tissue stripping.  The treatment of choice is surgical reduction of the acromioclavicular joint and some type of stabilization.  Treatment of type III injuries is controversial.

 

REFERENCES: Lemos MJ: The evaluation and treatment of the injured acromioclavicular joint in athletes.  Am J Sports Med 1998;26:137-144.

Rockwood CA Jr, Green DP, Bucholz RW, et al: Fractures in Adults, ed 5. Philadelphia, PA, Lippincott-Raven, 2001, pp 1209-1240.

 

50.       A 27-year-old professional baseball pitcher who underwent arthroscopic olecranon debridement continues to have medial-sided elbow pain during late cocking.  Physical examination reveals laxity and pain with valgus stress testing.  What is the most likely cause of his pain?

 

1-         Ulnar neuritis

2-         Excessive olecranon resection

3-         Osteochondritis dissecans of the capitellum

4-         Olecranon stress fracture

5-         Valgus extension overload

 

PREFERRED RESPONSE: 2

 

DISCUSSION: Both the medial collateral ligament and the olecranon contribute to valgus stability of the elbow.  Excessive olecranon resection increases the demand placed on the medial collateral ligament in resisting valgus forces during throwing.  Bone removal from the olecranon should be limited to osteophytes.

 

REFERENCES: Garrick JG (ed): Orthopaedic Knowledge Update: Sports Medicine 3.  Rosemont, IL, American Academy of Orthopaedic Surgeons, 2004, pp 101-111.

Kamineni S, Hirahara H, Pomianowski S, et al: Partial posteromedial olecranon resection:

A kinematic study.  J Bone Joint Surg Am 2003;85:1005-1011.

 

51.       A 22-year-old professional baseball catcher has posterior shoulder pain and severe external rotation weakness with the arm in adduction.  Radiographs are normal.  MRI scans are shown in Figures 15a through 15c.  Management should consist of

 

1-         aspiration and steroid injection.

2-         rest.

3-         acromioplasty.

4-         arthroscopic repair and decompression.

5-         rehabilitation.

 

PREFERRED RESPONSE: 4

 

DISCUSSION: The MRI scans reveal a large posterior paralabral cyst associated with a posterior-superior labral tear.  The cyst appears as a well-defined, smoothly marginated mass with low signal intensity on T1-weighted MRI scans and with high signal intensity on T2-weighted MRI scans.  MRI also reveals changes in the supraspinatus and infraspinatus muscles secondary to denervation, including decreased muscle bulk and fatty infiltration.  MRI has the added advantage, compared with other imaging modalities, of detecting intra-articular lesions, such as labral tears, which are frequently associated with ganglion cysts of the shoulder.  In this case of a professional baseball player with a space-occupying lesion causing nerve compression with an associated labral tear, the treatment of choice is arthroscopic decompression of the cyst and repair of the tear.  Acromioplasty would not address the primary pathology in this patient.

 

REFERENCES: Cummins CA, Messer TM, Nuber GW: Suprascapular nerve entrapment. 

J Bone Joint Surg Am 2000;82:415-424.

Martin SD, Warren RF, Martin TL, et al: Suprascapular neuropathy: Results of non-operative treatment.  J Bone Joint Surg Am 1997;79:1159-1165.

 

52.       A 21-year-old collegiate scholarship football player has an episode of transient quadriplegia.  An MRI scan of the cervical spine reveals cord edema and severe congenital spinal stenosis.  The athlete has aspirations of playing on a professional level and demands that he be allowed to play.  The team physician should give what recommendation to the college?

 

1-         Do not allow the athlete to return to football.

2-         Allow the athlete to participate.

3-         Allow the athlete to play only if he signs a waiver.

4-         Suggest that the college and athlete enter binding arbitration.

5-         Allow the athlete to play with special equipment.

 

PREFERRED RESPONSE: 1

 

DISCUSSION: Federal courts have ruled that a student-athlete does not have a constitutional right to participate in athletics against medical advice.  As long as the student retains his scholarship, the college is under no legal or ethical obligation to allow the student to participate in sports.  A waiver would not hold up in court and would not indemnify the college or the team physician against suit.  No equipment has been shown to be effective in preventing transient quadriplegia.

 

REFERENCES: Mathias MB: The competing demands of sport and health: An essay on the history of ethics in sports medicine.  Clin Sports Med 2004;23:195-214.

Bernstein J, Perlis C, Bartolozzi AR: Ethics in sports medicine.  Clin Orthop 2000;378:50-60.

 

53.       When performing a posterior cruciate ligament reconstruction with a tibial inlay-type approach, what is the approximate anatomic distance of the popliteal artery from the screws used for fixation of the bone block?

 

1-         5 mm

2-         10 mm

3-         15 mm

4-         20 mm

5-         25 mm

 

PREFERRED RESPONSE: 4

 

DISCUSSION: Miller and associates reported the results of a cadaveric study of the vascular risk of a posterior approach for posterior cruciate ligament reconstruction using the tibial inlay technique.  The average distance from the screw to the popliteal artery was 21.1 mm

(range, 18.1 mm to 31.7 mm).  Other approaches, such as the transtibial tunnel technique which involves drilling an anterior-posterior tunnel, have also been studied in cadavers.  Matava and associates noted that increasing flexion reduces but does not completely eliminate the risk of arterial injury during arthroscopic posterior cruciate ligament reconstruction.  However, this study did not use the small, medial utility incision recommended by Fanelli and associates, which creates an interval for the surgeon’s finger between the medial gastrocnemius and the posteromedial capsule so that any migration of the guidepin can be palpated and changed prior to any injury to the posterior neurovascular bundle.

 

REFERENCES: Matava MJ, Sethi NS, Totty WG: Proximity of the posterior cruciate ligament insertion to the popliteal artery as a function of the knee flexion angle: Implications for posterior cruciate ligament reconstruction.  Arthroscopy 2000;16:796-804.

Miller MD, Kline AJ, Gonzales J, et al: Vascular risk associated with posterior approach for posterior cruciate ligament reconstruction using the tibial inlay technique.  J Knee Surg 2002;15:137-140.

Johnson DH, Fanelli GC, Miller MD: PCL 2002: Indications, double-bundle versus inlay technique and revision surgery.  Arthroscopy 2002;18:40-52.

 

54.       Which of the following knee ligament injury patterns is most associated with an increase in external tibial rotation with the knee at 90 degrees of flexion? 

 

1-         Isolated tear of the posterior cruciate ligament

2-         Isolated tear of the lateral collateral ligament

3-         Combined tears of the posterior cruciate and lateral collateral ligaments

4-         Combined tears of the anterior cruciate and lateral collateral ligaments

5-         Combined tears of the lateral collateral and medial collateral ligaments

 

PREFERRED RESPONSE: 3

 

DISCUSSION: Cadaveric studies have shown that external rotation of the tibia is most pronounced following transection of the posterior cruciate and lateral collateral ligaments with the knee at 90 degrees of flexion.  Isolated release of the lateral collateral ligament results in increased external tibial rotation at 30 degrees. 

 

REFERENCES: Gollehon DL, Torzilli PA, Warren RF: The role of the posterolateral and cruciate ligaments in the stability of the human knee: A biomechanical study.  J Bone Joint Surg Am 1987;69:233-242.

Cooper DE: Tests for posterolateral instability of the knee in normal subjects: Results of examination under anesthesia.  J Bone Joint Surg Am 1991;73:30-36.

Veltri DM, Xeng XH, Torzilli PA, et al: The role of the cruciate and posterolateral ligaments in stability of the knee: A biomechanical study.  Am J Sports Med 1995;23:436-443.

 

55.       A 28-year-old professional dancer reports a 3-month history of progressive pain in the posterior aspect of the left ankle.  Her symptoms are worse when she assumes the en pointe position.  Examination reveals tenderness to palpation at the posterolateral aspect of the ankle posterior to the peroneal tendons which is made worse with passive plantar flexion.  There is no nodularity, fluctuance, or tenderness of the Achilles tendon.  The neurovascular examination is unremarkable.  A lateral radiograph and MRI scan are shown in Figures 16a and 16b, respectively.  Management should consist of

 

1-         a short leg cast with the ankle in slight plantar flexion.

2-         a corticosteroid injection into the retrocalcaneal bursa.

3-         excision of the os trigonum.

4-         excision of the superior tuberosity of the calcaneus.

5-         ankle arthroscopy with loose body removal.

 

PREFERRED RESPONSE: 3

 

DISCUSSION: The imaging studies reveal findings typical of the os trigonum syndrome.  This condition results from inflammation between the os trigonum and the adjacent talus.  The symptoms of posterior ankle pain are exacerbated by plantar flexion, which stresses the fibrous union between these two bones.  Definitive management of the high-level athlete involves excision of the os trigonum from a medial approach, although arthroscopic excision has also been described.  The os trigonum is not an intra-articular structure; therefore, ankle arthroscopy is neither diagnostic nor therapeutic.

 

REFERENCES: Abramowitz Y, Wollstein R, Barzilay Y, et al: Outcome of resection of a symptomatic os trigonum.  J Bone Joint Surg Am 2003;85:1051-1057.

Mouhsine E, Crevoisier X, Leyvraz P, et al: Post-traumatic overload or acute syndrome of the os trigonum: A possible cause of posterior ankle impingement.  Knee Surg Sports Traumatol Arthrosc 2004;12:250-253.

Wredmark T, Carlstedt CA, Bauer H, et al: Os trigonum syndrome: A clinical entity in ballet dancers.  Foot Ankle 1991;11:404-406.

 

56.       A professional pitcher reports pain localized to the medial aspect of his throwing elbow.  History reveals that he was pitching in a playoff game and heard and felt a pop in his elbow.  MRI reveals a complete ulnar-sided avulsion of the medial collateral ligament (MCL).  Examination reveals valgus instability and ulnar nerve involvement.  What recommendations should be made based on the patient’s desire to return to sport?

 

1-         Surgical reconstruction

2-         Rest, followed by physical therapy

3-         Splinting in 15 degrees of flexion

4-         Primary repair

5-         Arthroscopic debridement, followed by bracing in full extension for 4 weeks

 

PREFERRED RESPONSE: 1

 

DISCUSSION: Injuries to the MCL usually result from repetitive high valgus stress on the medial aspect of the elbow joint due to overhead throwing or racquet sports.  Excessive stresses during the late cocking and acceleration phase of throwing can injure the anterior band of the MCL.  Clinically, the injuries may present as chronic or acute, and a pop may be noted in the latter.  Associated ulnar nerve involvement is common.  Valgus instability is present in about 25% of patients.  Patients typically are athletes who participate in throwing and have localized medial elbow pain and tenderness along the course of a ligament that extends from the medial epicondyle of the distal humerus to the sublime tubercle of the ulna.  Surgical reconstruction is the procedure of choice in an athlete desiring a return to a high level of throwing. 

 

REFERENCES: Miller MD, Cooper DE, Warner JJP (eds): Review of Sports Medicine and Arthroscopy.  Philadelphia, PA, WB Saunders, 1995, p 230.

Arendt EA (ed): Orthopaedic Knowledge Update: Sports Medicine 2.  Rosemont, IL, American Academy of Orthopaedic Surgeons, 1999, p 227.

Altchek DW, Andrews JR (eds): The Athlete’s Elbow.  Philadelphia, PA, Lippincott Williams

& Wilkins, 2001, p 91.

57.       A 20-year-old collegiate football player who sustained blunt head trauma during the first half of a game is emotional and confused.  During the halftime intermission, his affect, memory, and disorientation are totally resolved and have returned to preinjury baseline.  The only residual finding is a very mild headache.  He wants to play the second half.  What is the most appropriate course of action?

 

1-         Permit him to play after a 15-minute cardiovascular challenge.

2-         Permit him to play after he satisfactorily completes “field-ready”
neuropsychiatric testing.

3-         Do not permit return to play until he can perform sport-specific skills.

4-         Do not permit return to play for 1 week.

5-         Do not permit return to play until all symptoms have resolved.

 

PREFERRED RESPONSE: 5

 

DISCUSSION: There is almost universal acceptance that an athlete may return to play after blunt head trauma only if he or she is totally asymptomatic.  Mild residual symptoms are considered an absolute contraindication for return to play.  Returning to play after a cardiovascular challenge or sport-specific activities is permitted on the pretext that the athlete is totally asymptomatic prior to these maneuvers.  Neuropsychiatric testing is being used more frequently to monitor residual cognitive effects after head trauma.  It has not been used as a return to play criterion.

 

REFERENCES: Garrick J (ed): Orthopaedic Knowledge Update: Sports Medicine 3. Rosemont, IL, American Academy of Orthopaedic Surgeons, 2004, pp 29-48.

Guskiewicz KM, McCrea, Marshall SW, et al: Cumulative effects associated with recurrent concussion in collegiate football players: The NCAA Concussion Study.  JAMA

2003;290:2549-2555.

 

58.       Which of the following actions best enhances performance when an athlete is participating in a 10K race?

 

1-         Load up on carbohydrates prior to the race.

2-         Hydrate adequately prior to the race.

3-         Replace fluid losses during the race.

4-         Replace calories burned during the race.

5-         Replace electrolytes lost during the race.

 

PREFERRED RESPONSE: 2

 

DISCUSSION: Proper hydration prior to an athletic event is the most important determinant of performance.  It is virtually impossible to keep pace with fluid loss during an athletic competition.  When a net loss of fluid occurs and the athlete is properly prehydrated, this fluid loss will not adversely affect performance.  It is not necessary to load up on carbohydrates prior to a 10K race, or to replace calories burned during the race.  Hyponatremia can develop in ultra-endurance athletes, especially marathoners, if they hydrate without replacing electrolytes lost through sweating; however, this is highly unlikely for a 10K race. 

 

REFERENCES: Newmark SR, Toppo FR, Adams G: Fluid and electrolyte replacement in the ultramarathon runner.  Am J Sports Med 1991;19:389-391.

Noakes T: Fluid replacement during marathon running.  Clin J Sports Med 2003;13:309-318.

 

59.       A 25-year-old competitive skier sustains a twisting injury to the right ankle while skiing.  She is unable to continue the activity secondary to severe lateral ankle pain.  Examination reveals ecchymosis and fullness over the lateral malleolus with pain and weakness on active ankle dorsiflexion and external rotation.  There is no medial-sided pain.  Neurovascular examination is normal.  An AP radiograph and MRI scan are shown in Figures 17a and 17b, respectively.  Management should consist of

 

1-         ice, elevation, and progressive weight bearing as tolerated.

2-         a walking boot for 6 weeks.

3-         a short leg non-weight-bearing cast for 6 weeks.

4-         temporary syndesmotic screw fixation.

5-         repair of the peroneal retinaculum.

 

PREFERRED RESPONSE: 5

 

DISCUSSION: The MRI scan shows a dislocated peroneus brevis tendon with disruption of the peroneal retinaculum.  This injury is commonly seen in skiers and is the result of peroneal contraction with the ankle everted and dorsiflexed.  Nonsurgical management is rarely successful; therefore, repair of the peroneal retinaculum is the treatment of choice.

 

REFERENCES: Eckert WR, Davis EA Jr: Acute rupture of the peroneal retinaculum.  J Bone Joint Surg Am 1976;58:670-672.

Murr S: Dislocation of the peroneal tendons with marginal fracture of the lateral malleolus. 

J Bone Joint Surg Br 1961;43:563-565.

Zoellner G, Clancy W Jr: Recurrent dislocation of the peroneal tendon.  J Bone Joint Surg Am 1979;61:292-294.

 

60.       Nonsurgical management of pectoralis major tears is likely to result in weakness
of glenohumeral

 

1-         abduction and external rotation.

2-         abduction and internal rotation.

3-         adduction and external rotation.

4-         adduction and internal rotation.

5-         external rotation and forward flexion.

 

PREFERRED RESPONSE: 4

 

DISCUSSION: Nonsurgical management is considered for proximal tears as well as partial tears in some individuals.  Surgical management is often not appropriate in older or sedentary patients.  However, patients treated nonsurgically will have a significant cosmetic defect, as well as weakness in adduction and internal rotation.

 

REFERENCES: Schepsis AA, Grafe MW, Jones HP, et al: Rupture of the pectoralis major muscle: Outcome or repair of acute and chronic injuries:  Am J Sports Med 2000;28:9-15.

Petilon J, Carr DR, Sekiya JK, et al: Pectoralis major muscle injuries: Evaluation and management.  J Am Acad Orthop Surg 2005;13:59-68.

 

61.        A 20-year-old man reports painless snapping about the lateral aspect of the right hip.  He denies any history of trauma.  Examination reveals no limp or tenderness.  Hip range of motion is full, and there is good strength.  Radiographs are normal.  What anatomic structure is most likely causing these symptoms?

 

1-         Acetabular labrum

2-         Iliopsoas

3-         Loose body

4-         Piriformis

5-         Iliotibial band

 

PREFERRED RESPONSE: 5

 

DISCUSSION: Coxa saltans (snapping hip syndrome) can occur in two forms: external/lateral or interior/medial/anterior.  This patient has the external/lateral form.  The external/lateral form involves the iliotibial band, tensor fascia, or gluteus medius, which snaps over the greater trochanter.  The external form usually can be treated with physical therapy alone; however, several recent studies report satisfactory results with surgical treatment.  Faraj and associates reported good results from surgical Z-plasty in a series of 10 patients.  White and associates reported good results in a series of 16 patients with 17 hips who underwent surgical release of an external snapping hip.  The interior/medial/anterior form can involve the iliopsoas tendon, acetabular labrum, subluxation of the hip, and loose bodies.

 

REFERENCES: White RA, Hughes MS, Burd T, et al: A new operative approach in the correction of external coxa saltans: The snapping hip.  Am J Sports Med 2004;32:1504-1508.

Faraj AA, Moulton A, Sirivastava VM: Snapping iliotibial band: Report of ten cases and review of the literature.  Acta Orthop Belg 2001;67:19-23.

Choi YS, Lee SM, Song BY, et al: Dynamic sonography of external snapping hip syndrome. 

J Ultrasound Med 2002;21:753-758.

 

62.       Which of the following statements correctly describes the results of gamma irradiation of musculoskeletal allograft?

 

1-         Exposure to 1.5 megarads effectively eliminates HIV.

2-         Exposure to 4 megarads does not affect graft mechanical properties.

3-         Gamma irradiation has been associated with chronic inflammation after implantation.

4-         Gamma irradiation is commonly used alone for sterilization.

5-         HIV cannot be eliminated with gamma irradiation alone without compromising the mechanical properties of the graft.

 

PREFERRED RESPONSE: 5

 

DISCUSSION: Low dose gamma irradiation (less than 3.0 megarads) with antibiotic soaks is one of the most common techniques for secondary sterilization.  Elimination of HIV with gamma irradiation requires doses estimated to be greater than 3.5 megarads.  Gamma irradiation levels of

4 megarads have been shown to alter the mechanical properties of human infrapatellar tendons.  Ethylene oxide, also used for allograft sterilization, has been associated with a chronic inflammatory process that resolved after graft removal. 

 

REFERENCES: Jackson DW, Windler GE, Simon TM: Intraarticular reaction associated with the use of freeze-dried, ethylene oxide-sterilized bone-patella tendon-bone allografts in the reconstruction of the anterior cruciate ligament.  Am J Sports Med 1990;18:1-10.

Conway B, Tomford W, Mankin HJ, et al: Radiosensitivity of HIV-1: Potential application to sterilization of bone allografts.  AIDS 1991;5:608-609.

Rasmussen TJ, Feder SM, Butler DL, et al:  The effects of 4 Mrad of gamma irradiation on the initial mechanical properties of bone-patellar tendon-bone grafts.  Arthroscopy 1994;10:188-197.

 

63.       A 35-year-old woman who is a recreational runner reports posterior knee pain and tightness in the knee with flexion during running.  She denies any history of trauma.  Examination reveals normal patellar glide and tilt and no patellar apprehension. 
Range of motion is 5 degrees to 120 degrees, and quadriceps function and knee ligamentous examination are normal.  Radiographs are normal.  An MRI scan is shown Figure 18.  What is the most likely diagnosis?

 

1-         Baker’s cyst

2-         Torn medial meniscus

3-         Patellofemoral pain syndrome

4-         Lipoma

5-         Ganglion cyst of the cruciates

 

PREFERRED RESPONSE: 5

 

DISCUSSION: Ganglia involving the cruciate ligaments have been recently reported as a cause of knee pain that interferes with knee flexion and extension.  The symptoms are poorly localized in this patient and not along the medial joint line, making the diagnosis of a torn medial meniscus less likely.  In addition, the MRI findings do not show a significant medial meniscal lesion.  A Baker’s cyst is usually posteromedial and extends posterior to the interval between the medial head of the gastrocnemius and semimembranosus.  MRI scans show a fluid-filled lesion with an increased signal on T1- and T2-weighted images.  A lipoma would be bright on the T1-weighted image only.

 

REFERENCES: Deutsch A, Veltri DM, Altchek DW, et al: Symptomatic intraarticular ganglia of the cruciate ligaments of the knee.  Arthroscopy 1994;10:219-223.

Brown MF, Dandy DJ: Intra-articular ganglia of the knee.  Arthroscopy 1990;6:322-323.

 

64.       A 12-year-old boy who pitches on two “select” baseball teams has had pain in his dominant right shoulder for the past 6 weeks.  The pain is present only with throwing and is associated with decreased throwing velocity and control.  He has no radiation of pain or paraesthesias of the upper extremity.  An AP radiograph and MRI scan are shown in Figures 19a and 19b, respectively.  Management should consist of

 

1-         rest from throwing activities.

2-         a subacromial corticosteroid injection.

3-         open reduction and internal fixation.

4-         arthroscopic labral repair.

5-         biopsy of the proximal humerus.

 

PREFERRED RESPONSE: 1

 

DISCUSSION: The imaging study demonstrates characteristics of Little Leaguer’s shoulder, including physeal widening.  This condition is secondary to overuse (typically throwing) and responds well to rest from the inciting activity.  There is no evidence from the patient’s history or examination that he has an impingement syndrome, nor is there any indication of labral pathology on the MRI scan.  The changes in the proximal humerus are classic for this condition and are not suggestive of a neoplastic process requiring biopsy for definitive diagnosis.

 

REFERENCES: Lipscomb AB: Baseball pitching injuries in growing athletes.  J Sports Med 1975;3:25-34.

Cahill BR, Tullos HS, Fain RH: Little league shoulder: Lesions of the proximal humeral epiphyseal plate.  J Sports Med 1974;2:150-152.

Barnett LS: Little league shoulder syndrome: Proximal humeral epiphysis in adolescent baseball pitchers: A case report.  J Bone Joint Surg Am 1985;67:495-496.

 

65.       An 18-year-old man underwent open reduction and internal fixation of a tibial spine avulsion and a posterolateral corner repair.  Two years later, he underwent lateral collateral ligament (LCL) and posterolateral corner reconstruction because of instability.  Examination reveals a pronounced lateral varus knee thrust when ambulating.  Varus stress in 30 degrees of flexion produces a 10-mm opening that is eliminated in extension.  The Lachman’s test is 2 mm with a firm end point, and the posterior drawer test is negative.  Standing radiographs show widening of the lateral joint space and a 5-degree mechanical varus alignment.  What is the most effective course of treatment?

 

1-         Physical therapy for quadriceps strengthening

2-         Functional bracing

3-         Anterior cruciate ligament (ACL) reconstruction

4-         Revision reconstruction of the LCL and posterolateral corner

5-         Valgus-producing high tibial osteotomy (HTO)

 

PREFERRED RESPONSE: 5

 

DISCUSSION: The patient has chronic posterolateral instability with a varus knee alignment; therefore, the most effective treatment is a valgus-producing HTO.  A repeat soft-tissue reconstruction without correction of the varus alignment will most likely fail.  An ACL reconstruction is not indicated with a normal Lachman’s test.  Physical therapy and bracing will have little effect.

 

REFERENCES: Naudie DD, Amendola A, Fowler PJ: Opening wedge high tibial osteotomy for symptomatic hyperextension-varus thrust.  Am J Sports Med 2004;32:60-70.

Covey DC: Injuries of the posterolateral corner of the knee.  J Bone Joint Surg Am

2001;83:106-118.

 

66.       As a baseball player dives to catch a line drive in the outfield, the ball strikes the tip of the player’s finger when extended, causing forcible flexion to avulse the extensor tendon from the distal phalanx.  Following evaluation and normal radiographic findings, initial management should include

 

1-         continuous extension splinting to the distal interphalangeal (DIP) joint for
6 weeks, followed by night splinting for an additional 6 weeks.

2-         splinting at the DIP joint in 20 degrees of flexion.

3-         percutaneous pinning.

4-         buddy taping.

5-         dynamic splinting for 8 weeks.

 

PREFERRED RESPONSE: 1

 

DISCUSSION: Avulsion of the terminal extensor tendon from the distal phalanx (mallet or baseball finger) may or may not be associated with a bony avulsion.  The injury is caused by forcible flexion of the DIP joint while catching a ball or hitting an object with the finger extended.  Most authorities recommend continuous extension splinting to the DIP joint for

6 weeks, followed by nighttime splinting for an additional 6 weeks.  It must be emphasized to the patient that at no time during the initial 6 weeks of treatment should the DIP joint be allowed to fall into flexion or an additional 6 weeks of continuous splinting is required.

 

REFERENCES: Miller MD, Cooper DE, Warner JP (eds): Review of Sports Medicine and Arthroscopy.  Philadelphia, PA, WB Saunders, 1995, p 255.

Rettig AC: Closed tendon injuries of the hand and wrist in the athlete.  Clin Sports Med 1992;11:77-99.

Arendt EA (ed): Orthopaedic Knowledge Update: Sports Medicine 2.  Rosemont, IL, American Academy of Orthopaedic Surgeons, 1999, pp 229-230.

 

67.       A favorable outcome following nonsurgical management of a partial tear of the posterior cruciate ligament (PCL) is best associated with

 

1-         hamstring strength.

2-         quadriceps strength.

3-         a body mass index of less than 30.

4-         anterior cruciate ligament stability.

5-         compliance with brace use.

 

PREFERRED RESPONSE: 2

 

DISCUSSION: Rehabilitation of the quadriceps muscle following a partial tear of the PCL has been associated with a favorable outcome.  The quadriceps acts an antagonist to the PCL because its contraction results in anterior tibial translation, which reduces the tensile stress on the injured ligament.  Strengthening of the hamstring musculature increases posterior tibial translation and is contraindicated during the early rehabilitative phase following a PCL injury.  Brace use has not been found to significantly alter the outcome following nonsurgical management of PCL tears.

 

REFERENCES: Parolie JM, Bergfeld JA: Long-term results of nonoperative treatment of isolated posterior cruciate ligament injuries in the athlete.  Am J Sports Med 1986;14:35-38.

Griffin JR, Annunziata CC, Harner CD: Posterior cruciate ligament injuries in the adult, in Drez D, DeLee JD, Miller MD (eds): Orthopaedic Sports Medicine Principles and Practice, ed 2.  Philadelphia, PA, WB Saunders, 2003, pp 2083-2106.

 

68.       A player on a professional football team sustains a knee injury and is diagnosed with an anterior cruciate ligament rupture.  When employed as the team physician, your ethical obligation is to inform

 

1-         the player but not the team.

2-         the team but not the player.

3-         neither the team nor the player.

4-         both the team and the player.

5-         the team, the player, and the media.

 

PREFERRED RESPONSE: 4

 

DISCUSSION: When you are employed as a team physician, you are obligated to inform the players and the team organization of all athletically relevant medical issues.  This differs significantly from the normal rule of patient confidentiality.  If the player came to see you and you were not the team physician, you may not inform the team unless the player so desires.  As the team physician, you are not obligated to inform the media. 

 

REFERENCES: Tucker AM: Ethics and the professional team physician.  Clin Sports Med 2004;23:227-241.

Johnson R: The unique ethics of sports medicine.  Clin Sports Med 2004;23:175-182.

 

69.       A 20-year-old basketball player reports a 6-month history of right groin pain that radiates into his testicles with activities of daily living.  He denies any history of trauma.  Examination reveals tenderness about the groin, and he has full hip range of motion.  The abdomen is soft.  Radiographs are normal.  Nonsurgical management has consisted of rest and physical therapy, but he continues to have pain.  What is the next step
in management?

 

1-         Bone scan

2-         CT

3-         High-dose steroids

4-         Sacroiliac joint injection

5-         Hernia repair

 

PREFERRED RESPONSE: 5

 

DISCUSSION: Sports hernias may be one of the most common causes of groin pain in athletes.  Resisted hip adduction is painful in the case of groin disruption.  Radiation of pain into the testicles and/or adductor region is often present.  Sports hernias are associated with weakening of the posterior inguinal wall.  In contrast with sports hernias, traditional or classic hernias can be readily detected on physical examination.  Diagnostic imaging studies are not helpful and only serve to help exclude other diagnoses.  Systemic high-dose steroids or sacroiliac joint injections have no role in treatment.  High success rates have been reported for laparoscopic hernia repair in athletes.

 

REFERENCES: Kluin J, den Hoed PT, van Linschoten R, et al: Endoscopic evaluation and treatment of groin pain in the athlete.  Am J Sports Med 2004;32:944-949.

Genitsaris M, Goulimaris I, Sikas N: Laparoscopic repair of groin pain in athletes.  Am J Sports Med 2004;32:1238-1242.  

Meyers WC, Foley DP, Garrett WE, et al: Management of severe lower abdominal or inguinal pain in high-performance athletes: PAIN (Performing Athletes with Abdominal or Inguinal Neuromuscular Pain Study Group).  Am J Sports Med 2000;28:2-8.

 

70.       A 45-year-old tennis player undergoes surgery for chronic lateral epicondylitis.  After returning to play, he notes increasing lateral elbow pain with mechanical catching and locking.  Examination shows positive supine posterolateral rotatory instability.  What ligament has been injured?

 

1-         Annular

2-         Anterior band of the medial collateral

3-         Lateral orbicular

4-         Lateral radial collateral

5-         Lateral ulnar collateral

 

PREFERRED RESPONSE: 5

 

DISCUSSION: The patient has sustained an iatrogenic injury to the lateral ulnar collateral ligament.  This injury has been reported after lateral approaches to the elbow.  The orbicular, annular, and lateral radial collateral ligaments have a much less important role in lateral elbow stability.  The anterior band of the ulnar collateral ligament is on the medial side of the elbow and is important for valgus stability.

 

REFERENCES: O’Driscoll SW, Bell DF, Morrey BF: Posterolateral rotatory instability of the elbow.  J Bone Joint Surg Am 1991;73:440-446.

O’Driscoll SW, Morrey BF: Surgical reconstruction of the lateral collateral ligament, in Morrey BF (ed): The Elbow.  Philadelphia, PA, Lippincott, Williams and Wilkins, 1994, pp 169-182.

 

71.        A female cross-country runner has an insidious onset of right groin pain.  Radiographs of the right hip reveal a tension-side stress fracture.  History reveals that she was treated for a “foot” fracture 1 year ago.  In addition to performing internal fixation of the femoral neck, which of the following should be obtained?

 

1-         Menstrual history

2-         Family history

3-         Serum calcium level

4-         MRI of the hip

5-         Contralateral hip radiograph

 

PREFERRED RESPONSE: 1

 

DISCUSSION: Stress fractures in female long distance runners are frequently associated with the Female Athletic Triad.  The triad consists of osteoporosis, amenorrhea, and altered eating habits.  A thorough menstrual history, including age of menarche, history of amenorrhea, and use of oral contraceptives, is imperative.  Amenorrhea leads to osteoporosis and predisposes the athlete to fractures.  An MRI of the hip is not necessary because a fracture is evident on the radiograph.  Serum calcium levels are normal in osteoporosis, a family history would be noncontributory, and it is highly unlikely that a contralateral hip radiograph will yield useful information.

 

REFERENCES: Bennell KL, Malcolm SA, Thomas SA, et al: Risk factors for stress fractures in track and field athletes: A twelve-month prospective study.  Am J Sports Med 1996;24:810-818.

Barrow GW, Saha H: Menstrual irregularity and stress fractures in collegiate female distance runners.  Am J Sports Med 1988;16:209-216.

 

72.       An 18-year-old gymnast has had a 1-year history of foot pain.  Examination reveals medial midfoot tenderness without swelling.  Non-weight-bearing in a cast for 6 weeks has failed to provide relief.  An axial CT scan of the midfoot is shown in Figure 20.  What is the optimal treatment for this condition?

 

1-         Partial weight bearing in a walking cast for an additional 6 weeks

2-         Open reduction and internal fixation

3-         Open reduction and internal fixation with autologous bone grafting

4-         No treatment

5-         Non-weight-bearing in a cast for an additional 6 weeks

 

PREFERRED RESPONSE: 3

 

DISCUSSION: Stress fractures of the navicular are often seen in running and jumping sports.  Whereas most individuals heal with nonsurgical management consisting of 6 weeks of casting, this gymnast has had pain for 1 year and nonsurgical management has failed.  Open reduction with bone grafting is the preferred treatment.

 

REFERENCES: Quirk RM: Stress fractures of the navicular.  Foot Ankle Int 1998;19:494-496.

Saxena A, Fullem B, Hannaford D: Results of treatment of 22 navicular stress fractures and a new proposed radiographic classification system.  J Foot Ankle Surg 2000;39:96-103.

 

73.       A 20-year-old soccer player who collapsed after a goal kick reports weakness and nausea.  He appears slightly confused.  Examination reveals that he is not sweating.  His skin is warm and dry.  The outdoor temperature is 80 degrees F (26.6 degrees C) with a relative humidity of 80%.  Management should consist of

 

1-         a drink of water.

2-         a sports drink with electrolytes.

3-         placement in the reverse Trendelenburg position in a shaded area.

4-         immersion in a warm water bath.

5-         transportation to the emergency department.

 

PREFERRED RESPONSE: 5

 

DISCUSSION: There is a spectrum of heat-related conditions.  Heat cramps are the mildest form of heat illness.  In heat exhaustion, cramps are associated with headache and weakness, and the skin is pale and moist.  Treatment of heat cramps or heat exhaustion consists of removing and loosening excess clothing, applying ice to the axilla and groin, ingestion of cool water, and cool water sprays.  This patient demonstrates symptoms of heat stroke which is a medical emergency.  The core body temperature may be as high as 106 to 110 degrees F (41.1 to 43.3 degrees C).  In heat stroke, the patient may no longer be sweating, and the skin may be hot and red.  The athlete is usually confused, weak, nauseated, and may have seizure activity.  Central nervous system depression has been called the most important marker of heat stroke, and progresses from confusion and bizarre behavior to collapse, delirium, and coma.  Bizarre behavior is often the first sign of heat stroke.  The patient needs to be treated and moved to a medical facility rapidly.  During transfer, IV fluids and cooling of the athlete should be initiated.  The best treatment of heat-related illness appears to be prevention with adequate hydration and monitoring of conditions (temperature and humidity), with cancellation of competition when conditions do not comply with guidelines. 

 

REFERENCES: Griffin LY: Emergency preparedness: Things to consider before the game starts.  J Bone Joint Surg Am 2005;87:894-902.

Barker TA, Motz HA, Gersoff WK: Environmental factors in athletic performance, in Fu FH, Stone DA (eds): Sports Injuries, ed 2.  Philadelphia, PA, Lippincott, 2001, pp 67-68.

Roberts WO: Environmental concerns, in Kibler WB (ed): ACSM’s Handbook for the Team Physician.  Baltimore, MD, Williams & Wilkins, 1996, p 172.

 

74.       What is the most accurate description of the relationship between gender and knee loading during landing while playing basketball?

 

1-         Males have greater total valgus knee loading.

2-         Females have greater total valgus knee loading.

3-         Males have greater total varus knee loading.

4-         Females have greater total varus knee loading.

5-         There is no gender difference in total varus or valgus knee loading.

 

PREFERRED RESPONSE: 2

DISCUSSION: Ford and associates studied 81 high school basketball players and found that females landed with greater total valgus knee loading and a greater maximum valgus knee angle than male athletes.  Hewett and associates reported in a study of 205 female athletes that those with increased dynamic valgus and high abduction loads were at increased risk of anterior cruciate ligament injury. 

 

REFERENCES: Hewett TE, Myer GD, Ford KR, et al: Biomechanical measures of neuromuscular control and valgus loading of the knee predict anterior cruciate ligament injury risk in female athletes: A prospective study.  Am J Sports Med 2005;33:492-501.

Ford KR, Meyer GD, Hewett TE: Valgus knee motion during landing in high school female and male basketball players.  Med Sci Sports Exer 2003;35:1745-1750.

 

75.       What is the most common cause of the new onset of amenorrhea in a female endurance athlete who is not sexually active?

 

1-         Insufficient caloric intake

2-         Physical stress

3-         Use of oral contraceptives

4-         Diabetes mellitus

5-         Chromosomal abnormalities

 

PREFERRED RESPONSE: 1

 

DISCUSSION: Insufficient caloric intake caused by either a poor diet or an eating disorder is the most common cause for the loss of menses in a female athlete.  In the face of adequate caloric intake, stress is unlikely to cause amenorrhea.  Oral contraceptives control menses but do not eliminate it.  Diabetes mellitus does not cause the new onset of amenorrhea.  Pregnancy can be a cause in a sexually active athlete.  Chromosomal abnormalities can result in delayed or absent menarche but not the onset of amenorrhea in a postmenarchal female.

 

REFERENCES: Constantini NW: Clinical consequences of amenorrhea.  Sports Med 1994;17:213-223.

Bennell KL, Malcolm SA, Thomas SA, et al: Risk factors for stress fractures in track and field athletes: A twelve-month prospective study.  Am J Sports Med 1996;24:810-818.

 

76.       Closure of the rotator cuff interval results in elimination of which direction of
shoulder instability? 

 

1-         Posterosuperior

2-         Posteroinferior

3-         Anterosuperior

4-         Anteroinferior

5-         Multidirectional

 

PREFERRED RESPONSE: 2

 

DISCUSSION: The rotator cuff interval consists of the superior glenohumeral and coracohumeral ligaments.  Injury to this ligament complex leads to posteroinferior shoulder instability.  Tightening of these tissues through surgical means has been shown to result in a significant reduction in posteroinferior translation of the humerus in relation to the glenoid.

 

REFERENCES: Harryman DT II, Sidles JA, Harris SL, et al: The role of the rotator interval capsule in passive motion and stability of the shoulder.  J Bone Joint Surg Am 1992;74:53-66.

O’Brien SJ, Schwartz RS, Warren RF, et al: Capsular restraints to anterior-posterior motion of the abducted shoulder: A biomechanical study.  J Shoulder Elbow Surg 1995;4:298-308.

Warner JJ, Deng XH, Warren RF, et al: Static capsuloligamentous restraints to superior-inferior translation of the glenohumeral joint.  Am J Sports Med 1992;20:675-685.

 

77.       In overhead athletic activities, the kinetic chain generates what percentage of force from the leg and trunk segments of the chain?

 

1-         20% to 30%

2-         30% to 40%

3-         50% to 60%

4-         60% to 70%

5-         80% to 90%

 

PREFERRED RESPONSE: 3

 

DISCUSSION: The leg and trunk provide a stable base for arm motion, supply rotational momentum for force generation, and generate 50% to 55% of the total force and kinetic energy in the tennis serve.

 

REFERENCES: Garrick JG (ed): Orthopaedic Knowledge Update: Sports Medicine 3.  Rosemont, IL, American Academy of Orthopaedic Surgeons, 2004, p 47.

McClure PW, Michener LA, Sennett BJ, et al: Direct 3-dimensional measurement of scapular kinematics during dynamic movements in vivo.  J Shoulder Elbow Surg 2001;10:269-277.

Burkhart SS, Morgan CD, Kibler WB: The disabled throwing shoulder: Spectrum of pathology. Part III: The SICK scapula, scapular dyskinesis, the kinetic chain, and rehabilitation.  Arthroscopy 2003;19:641-661.

 

78.       A 40-year-old right-handed professional football player reports persistent right
wrist pain after falling during a game 5 days ago.  A radiograph is shown in
Figure 21.  Management should consist of

 

1-         immobilization in a short arm thumb spica cast.

2-         immobilization in a long arm thumb spica cast.

3-         arthroscopic repair and percutaneous pinning.

4-         open repair and percutaneous pinning.

5-         dorsal capsulodesis.

 

PREFERRED RESPONSE: 4

 

DISCUSSION: The radiograph reveals an increased distance between the scaphoid and the lunate, which is indicative of scapholunate disassociation.  A ring sign is also present, which represents the distal pole of the scaphoid viewed end on in a palmarly flexed position.  In the acute setting, the scapholunate can be repaired.  Open repair and percutaneous pinning is the treatment of choice.  Dorsal capsulodesis is performed in the chronic setting if such an injury is initially missed.

 

REFERENCES: Cohen MS: Ligamentous injuries of the wrist in the athlete.  Clin Sports Med 1998;17:533-552.

Taleisnik J: Carpal instability.  J Bone Joint Surg Am 1988;70:1262-1268.

 

79.       A 29-year-old ultramarathoner, who is halfway into a 50-mile race, is sweating profusely.  He suddenly collapses, is unresponsive, and has violent muscle contractions.  Prior to these symptoms, he had been drinking water at every water stop (every 1 mile).  What is the most likely diagnosis?

 

1-         Hypernatremia

2-         Hyponatremia

3-         Hyperkalemia

4-         Hypokalemia

5-         Hyperuremia

 

PREFERRED RESPONSE: 2

 

DISCUSSION: Hyponatremia (“water intoxication”) can occur in endurance athletes such as ultramarathoners who are sweating profusely and drinking only water as fluid replacement.  Sports drinks which contain electrolytes are a better replacement in this group of athletes.  Sodium is the mineral most commonly affected by physical exercise.  Sodium concentration in sweat depends on diet, hydration, and heat acclimation.  In most cases, sodium lost in sweat can be replaced by regular diet.  Potassium plays an important role in nerve conduction and muscle contraction but is not lost in excessive amounts in sweat during exercise.  The most frequent loss of potassium is through gastrointestinal disorders or excessive loss from the kidneys.  Rehrer reported that overhydrating during very long-lasting exercise in the heat with low or negligible sodium intake can result in reduced performance and hyponatremia.  With hyponatremia, the serum sodium is abnormally low, resulting in brain swelling, seizures, coma, and potentially death.  Interestingly, hyponatremia is rarely seen in adolescent athletes and young children.

 

REFERENCES: Griffin LY: Emergency preparedness: Things to consider before the game starts.  J Bone Joint Surg Am 2005;87:894-902.

Rehrer NJ: Fluid and electrolyte balance in ultra-endurance sport.  Sports Med 2001;31:701-715.

 

80.       A 12-year-old Little League pitcher reports lateral elbow pain and “catching.”  Examination reveals painful pronation and supination and tenderness over the lateral elbow.  Radiographs are shown in Figures 22a and 22b.  Initial management should consist of

 

1-         rest and repeat examination and radiographs until complete healing occurs.

2-         rest and resumption of play when he is asymptomatic and examination shows restoration of painless range of motion.

3-         arthroscopic in situ drilling.

4-         arthroscopic drilling and internal fixation.

5-         arthroscopy with removal of the loose body, followed by lateral column osteotomy.

 

PREFERRED RESPONSE: 2

 

DISCUSSION: Osteochondritis of the capitellum is a common problem in young throwing athletes and gymnasts.  The mechanism of injury involves lateral compression and axial loading of the capitellum.  Repetitive trauma causes ischemia with resultant osteochondral necrosis and sometimes eventual separation.  Initial management includes rest for a minimum of 6 weeks; occasionally bracing is used.  At long-term follow-up, there is typically an observed radiographic abnormality indicating incomplete healing even in asymptomatic patients.  Arthroscopy with in situ drilling is reserved for symptomatic lesions that have an intact articular surface.  Lesions with partial separation often require fixation.  Lateral column osteotomy is a new investigational procedure designed to relieve lateral compression forces and may be used in salvage cases.

 

REFERENCES: Kobayashi K, Burton KJ, Rodner C, et al: Lateral compression injuries in the pediatric elbow: Panner’s disease and osteochondritis dissecans of the capitellum.  J Am Acad Orthop Surg 2004;12:246-254.

Yadao MA, Field LD, Savoie FH III: Osteochondritis dissecans of the elbow.  Instr Course Lect 2004;53:599-606.

81.       Medial dislocation of the long head of the biceps tendon in the shoulder is most commonly caused by a

 

1-         tear of the subscapularis tendon.

2-         tear of the supraspinatus tendon.

3-         tear of the transverse ligament.

4-         type I SLAP tear.

5-         congenitally shallow bicipital groove.

 

PREFERRED RESPONSE: 1

 

DISCUSSION: Medial dislocation of the biceps tendon in the shoulder is commonly associated with subscapularis tendon tears.  Although type II SLAP tears can result in bicipital instability, type I SLAP lesions do not.  Congenitally shallow grooves and tears of the transverse ligaments usually do not lead to dislocation of the biceps tendon.  Supraspinatus tendon tears are associated with long head of the biceps tendon ruptures but do not cause biceps tendon dislocations.

 

REFERENCES: Werner A, Mueller T, Boehm D, et al: The stabilizing sling for the long head of the biceps tendon in the rotator cuff interval: A histoanatomic study.  Am J Sports Med 2000;28:28-31.

Arendt EA (ed): Orthopaedic Knowledge Update: Sports 2.  Rosemont, IL, American Academy of Orthopaedic Surgeons, 1999, pp 165-189.

 

82.       Tension force in the anterior cruciate ligament during passive range of motion is
highest at

 

1-         full extension.

2-         30 degrees of flexion.

3-         60 degrees of flexion.

4-         90 degrees of flexion.

5-         120 degrees of flexion.

 

PREFERRED RESPONSE: 1

 

DISCUSSION: Tension forces in the healthy, as well as the reconstructed, anterior cruciate ligament were measured and found to be highest with the knee in full extension and decreased as the flexion increased.

 

REFERENCES: Markolf KL, Burchfield DM, Shapiro MM, et al: Biomechanical consequences of replacement of the anterior cruciate ligament with a patellar ligament allograft.  Part II: Forces in the graft compared with forces in the intact ligament.  J Bone Joint Surg Am 1996;78:1728-1734.

Beynnon BD, Johnson RJ, Fleming BC, et al: The measurement of elongation of anterior cruciate-ligament grafts in vivo.  J Bone Joint Surg Am  1994;76:520-531.

 

83.       Compared to eumenorrheic athletes, amenorrheic athletes have more frequent occurrences of

 

1-         stress fractures.

2-         scoliosis.

3-         pes planus.

4-         meniscal tears.

5-         ankle sprains.

 

PREFERRED RESPONSE: 1

 

DISCUSSION: In secondary amenorrhea, women do not receive the estrogen needed to maintain adequate bone mineralization.  This hypoestrogenic state affects bone density, and there is evidence that stress fractures are more frequent in amenorrheic than eumenorrheic athletes.  The other conditions are not seen with increased frequency in amenorrheic athletes.

 

REFERENCES: Warren MP: Health issues for women athletes: Exercise-induced amenorrhea. 

J Clin Endocrinol Metab 1999;84:1892-1896.

Rencken ML, Chesnut CH III, Drinkwater BL: Bone density at multiple skeletal sites in amenorrheic athletes.  JAMA 1996;276:238-240.

 

84.       Figure 23 shows the postoperative radiograph of a patient who underwent an anterior cruciate ligament (ACL) reconstruction (with bone-patella tendon-bone autograft) that failed.  He initially had loss of flexion postoperatively.  What is the most likely cause of this failure?

 

1-         Fixation in the tibial tunnel

2-         Fixation in the femoral tunnel

3-         Posterior placement of the tibial tunnel

4-         Anterior placement of the femoral tunnel

5-         Size of the patellar autograft

 

PREFERRED RESPONSE: 4

 

DISCUSSION: The key to this question is the fact that the patient initially lost flexion postoperatively and this relates to anterior placement of the femoral tunnel, thus capturing the knee.  The bone plug seen on the radiograph is actually from the tibial tunnel, but this occurred as the patient forced flexion until failure of the ACL graft and pullout of the plug from the tunnel.  Although it could be argued that better tibial fixation would have prevented this failure, poor placement of the femoral tunnel led to the failure of this ACL reconstruction.

 

REFERENCES: Fu FH, Bennett CH, Latterman C, et al: Current trends in anterior cruciate ligament reconstruction: Part 1.  Biology and biomechanics of reconstruction.  Am J Sports Med 1999;27:821-830.

Fu FH, Bennett CH, Ma CB, et al: Current trends in anterior cruciate ligament reconstruction: Part II.  Operative procedures and clinical correlations.  Am J Sports Med 2000;28:124-130.

85.       A 22-year-old man reports anterior knee pain, swelling, and is unable to perform a straight leg raise after undergoing endoscopic anterior cruciate ligament (ACL) reconstruction with a bone-patellar tendon-bone autograft 1 week ago.  He is afebrile.  Examination reveals a clean incision, moderate effusion, a weak isometric quadriceps contraction, active knee range of motion of 5 degrees to 45 degrees, and the patella is ballottable.  Knee radiographs show postoperative changes with good femoral and tibial tunnel placements, and normal patellar height.  What is the next most appropriate step
in management?

 

1-         Electromyography (EMG) and nerve conduction velocity studies (NCVS)

2-         Diagnostic ultrasonography of the patellar tendon

3-         MRI

4-         Continuous passive motion

5-         Knee aspiration

 

PREFERRED RESPONSE: 5

 

DISCUSSION: Knee pain and swelling in the first week after ACL reconstruction is usually related to a postoperative hemarthrosis.  A large hemarthrosis creates capsular distension, which inhibits active quadriceps contraction by a neurologic reflex, the H. reflex.  Kennedy and associates reported that an experimentally induced knee effusion at 60 mL was found to result in profound inhibition of reflexly evoked quadriceps contraction.  Removal of the hemarthrosis by aspiration will improve strength and often instantaneously restore the ability to contract the quadriceps muscle.  A large effusion will also limit knee flexion.  EMG and NCVS are not necessary unless there is a high index of suspicion of a femoral neuropathy.  Diagnostic ultrasonography is not necessary in this patient but can be useful in the assessment of patellar tendon integrity.  MRI is not indicated and would most likely be limited by artifact and postoperative changes.  Continuous passive motion is not indicated and would most likely worsen the patient’s symptoms.

 

REFERENCES: Kennedy JC, Alexander IJ, Hayes KC: Nerve supply of the human knee and its functional importance.  Am J Sports Med 1982;10:329-335.

Fahrer H, Rentsch HU, Gerber NJ, et al:  Knee effusion and reflex inhibition of the quadriceps: A bar to effective retraining.  J Bone Joint Surg Br 1988;70:635-638.

 

86.       A 46-year-old woman fell from her bicycle and sustained the injury shown in
Figure 24.  Which of the following ligaments has been disrupted?

 

1-         Acromioclavicular

2-         Acromioclavicular and coracoclavicular

3-         Coracoclavicular

4-         Coracoacromial and sternoclavicular

5-         Sternoclavicular

 

PREFERRED RESPONSE: 2

 

DISCUSSION: The radiograph shows a type V acromioclavicular joint injury.  Type V injuries involve disruption of the acromioclavicular and coracoclavicular ligaments.  Type I injuries involve a sprain of the acromioclavicular joint ligaments.  Type II injuries involve disruption of the acromioclavicular joint ligaments; the coracoclavicular ligaments are partially injured.  Sternoclavicular ligaments stabilize the medial clavicle and the sternum; they are not damaged with acromioclavicular joint dislocations.

 

REFERENCES: Fukuda K, Craig EV, An KN, et al: Biomechanical study of the ligamentous system of the acromioclavicular joint.  J Bone Joint Surg Am 1986;68:434-439.

Bosworth B: Complete acromioclavicular dislocation.  N Engl J Med 1949;241:221-225.

 

87.       The use of knee arthroscopy following total knee arthroplasty is most effective in treating which of the following conditions?

 

1-         Patellar clunk syndrome

2-         Septic arthritis

3-         Nonspecific pain

4-         Improper tracking of the patellar component

5-         Synovitis secondary to polyethylene wear

 

PREFERRED RESPONSE: 1

 

DISCUSSION: Patellar clunk syndrome is associated with certain types of posterior stabilized knee arthroplasties.  Arthroscopic resection of the band of inflammatory tissue inferior to the patellar component is effective in treating this condition.  Arthroscopic lavage of infected knee arthroplasties is not associated with an acceptable success rate.  Diagnostic arthroscopy for nonspecific pain following arthroplasty is not uniformly successful.  Patellar component maltracking is frequently associated with component malposition and is not alleviated by an arthroscopic lateral release.  Synovitis secondary to polyethylene wear is best treated by exchange of the polyethylene spacer and not arthroscopic synovectomy.

 

REFERENCES: Lucas TS, DeLuca PF, Nazarian DG, et al: Arthroscopic treatment of patellar clunk.  Clin Orthop 1999;367:226-229.

Takahashi M, Miyamoto S, Nagano A: Arthroscopic treatment of soft-tissue impingement under the patella after total knee arthroplasty.  Arthroscopy 2002;18:E20.

 

88.       Significant anterior tibial translation occurs during which of the following
rehabilitation exercises?

 

1-         Terminal weight-bearing knee extension

2-         Terminal non-weight-bearing knee extension

3-         Terminal weight-bearing knee flexion

4-         Terminal non-weight-bearing knee flexion

5-         Mid-range weight-bearing knee flexion

 

PREFERRED RESPONSE: 2

 

DISCUSSION: Terminal non-weight-bearing knee extension exercises from 60 degrees

to 0 degrees of flexion increase anterior tibial translation.  It is for this reason that this type of exercise should be avoided in the early phase of rehabilitation following anterior cruciate ligament reconstruction so as not to place a tensile strain on the graft.  The other rehabilitation exercises either lead to posterior tibial translation in relation to the femur or have no significant effect on tibial translation.

 

REFERENCES: Grood ES, Suntay WJ, Noyes FR, et al: Biomechanics of the knee

extension exercise: Effect of cutting the anterior cruciate ligament.  J Bone Joint Surg Am 1984;66:725-734.

Lutz GE, Palmitier RA, An KN: Comparison of tibiofemoral joint forces during open-kinetic-chain and closed-kinetic-chain exercises.  J Bone Joint Surg Am 1993;75:732-739.

Wilk KE, Escamilla RF, Fleisig GS, et al: A comparison of tibiofemoral joint forces and electromyographic activity during open and closed kinetic chain exercises.  Am J Sports Med 1996;24:518-527.

 

89.       A 22-year-old volleyball player reports the insidious onset of superior and posterior shoulder pain.  Radiographs are normal.  An MRI scan is shown in Figure 25.  What is the most specific physical examination finding?

 

1-         Positive impingement sign

2-         Positive apprehension

3-         Positive active compression

4-         Weakness of external rotation

5-         Weakness of abduction

 

PREFERRED RESPONSE: 4

 

DISCUSSION: Overhead athletes are prone to a number of problems involving the shoulder.  Pitchers and volleyball players are susceptible to posterior superior labral tears and internal impingement.  These patients will have posterior superior shoulder pain, a positive relocation sign, and a positive active compression test.  Occasionally, these posterior superior labral tears are associated with a spinoglenoid cyst as seen in the MRI scan.  These cysts cause compression of the suprascapular nerve which manifests primarily as weakness of the infraspinatus, resulting in weakness of external rotation. 

REFERENCES: Romeo AA, Rotenberg DD, Bach BR Jr: Suprascapular neuropathy.  J Am Acad Orthop Surg 1999;7:358-367.

Cummins CA, Messer TM, Nuber GW: Suprascapular nerve entrapment.  J Bone Joint Surg Am 2000;82:415-424.

 

90.       A 20-year-old male lacrosse player sustains an anterior dislocation of the shoulder.  He is extremely concerned about recurrent dislocations.  Which of the following treatments has been shown to reduce the risk of recurrent dislocation?

 

1-         Functional rehabilitation and return to play when he has pain-free range of motion

2-         Immobilization in internal rotation for 6 weeks

3-         Immobilization in internal rotation for 3 weeks, followed by 3 weeks of supervised rehabilitation

4-         Immobilization with the arm in neutral rotation

5-         Immobilization with the arm in 30 degrees of external rotation

 

PREFERRED RESPONSE: 5

 

DISCUSSION: Recent evidence has shown that the position of immobilization of the shoulder after a dislocation influences the reduction of the Bankart lesion.  In an MRI study in patients who sustained an anterior dislocation, the Bankart lesion was reduced to the glenoid anatomically with the arm in 30 degrees of external rotation.  Subsequently, a clinical follow-up study has shown a reduction in recurrence rates when the arm is immobilized in external rotation compared to internal rotation.   

 

REFERENCES: Itoi E, Hatakeyama Y, Kido T, et al: A new method of immobilization after traumatic anterior dislocation of the shoulder: A preliminary study.  J Shoulder Elbow Surg 2003;12:413-415.

Itoi E, Sashi R, Minagawa H, et al: Position of immobilization after dislocation of the glenohumeral joint: A study with use of magnetic resonance imaging.  J Bone Joint Surg Am 2001;83:661-667.

 

91.        A 43-year-old soccer player who had knee pain following a twisting injury underwent an arthroscopic meniscectomy 6 months ago.  He continues to report posterior knee pain.  Examination reveals soft-tissue fullness and tenderness just above the popliteal fossa, trace knee effusion, full range of knee motion, no instability, and negative meniscal signs.  Radiographs show some mild medial joint space narrowing but no other bony changes.  What is the next most appropriate step in management?

 

1-         Corticosteroid injection

2-         MRI

3-         Bone scan

4-         Unloader brace

5-         Nonsteroidal anti-inflammatory drugs

 

PREFERRED RESPONSE: 2

 

DISCUSSION: The phenomenon of tumors misdiagnosed as athletic injuries has been termed “sports tumors.”  Lewis and Reilly presented a series of 36 patients who initially were thought to have a sports-related injury but ultimately were diagnosed with a primary bone tumor, soft-tissue tumor, or tumor-like condition.  Muscolo and associates presented a series of 25 tumors that had been previously treated with an intra-articular procedure as a result of a misdiagnosis of an athletic injury.  Initial diagnoses included 21 meniscal lesions, one traumatic synovial cyst, one patellofemoral subluxation, one anterior cruciate ligament tear, and one case of nonspecific synovitis.  The final diagnoses were a malignant tumor in 14 patients and a benign tumor in

11 patients.  The authors noted that oncologic surgical treatment was affected in 15 of the

25 patients.  The most frequent causes of erroneous diagnosis were initial poor quality radiographs and an unquestioned original diagnosis despite persistent symptoms.  Persistent symptoms warrant further diagnostic studies, not additional treatment such as physical therapy, corticosteroid injection, or an unloader brace.  Although a bone scan may be helpful in this case and confirm arthrosis of the medial compartment, the suspicion of a soft-tissue mass makes MRI the imaging modality of choice.

 

REFERENCES: Muscolo DL, Ayerza MA, Makino A, et al: Tumors about the knee misdiagnosed as athletic injuries.  J Bone Joint Surg Am 2003;85:1209-1214.

Lewis MM, Reilly JF: Sports tumors.  Am J Sports Med 1987;15:362-365.

 

92.       Figures 26a through 26c show the MRI scans of a 47-year-old man who underwent arthroscopic shoulder surgery 6 months ago and continues to have pain despite a prolonged course of rehabilitation.  Management should now consist of

 

1-         rotator cuff repair.

2-         revision acromioplasty.

3-         fragment excision.

4-         open reduction and internal fixation.

5-         continued rehabilitation.

 

PREFERRED RESPONSE: 4

DISCUSSION: The MRI scans show an os acromiale of the mesoacromion type.  This represents an unfused acromial apophysis.  Pain is thought to be caused by either motion at the site or downward displacement of the anterior aspect of the acromion onto the rotator cuff, causing impingement.  Most patients can be treated nonsurgically as they are usually asymptomatic.  In those patients with persistent symptoms of pain and tenderness over the acromion, surgery consisting of rigid internal fixation and bone grafting has yielded satisfactory results.  Excision may be a viable treatment option for the preacromion type.

 

REFERENCES: Herzog RJ: Magnetic resonance imaging of the shoulder.  Instr Course Lect 1998;47:3-20.

Warner JP, Beim GM, Higgins L: The treatment of symptomatic os acromiale.  J Bone Joint Surg Am 1998;80:1320-1326.

Sammarco VJ: Os acromiale: Frequency, anatomy, and clinical implications.  J Bone Joint Surg Am 2000;82:394-400.

 

93.       An 18-year-old rugby player has had pain in his ring finger after missing a tackle 1 week ago.  Examination reveals tenderness in the distal palm, and he is unable to actively flex the distal interphalangeal (DIP) joint.  Radiographs are normal.  What is the most appropriate management?

 

1-         Acute tendon repair

2-         DIP joint extension splinting for 6 weeks

3-         DIP and proximal interphalangeal joint extension splinting for 6 weeks

4-         Buddy taping to the middle finger for 2 weeks

5-         Early range-of-motion exercises and return to play as pain permits

 

PREFERRED RESPONSE: 1

 

DISCUSSION: Flexor digitorum profundus rupture or “rugger jersey finger” often occurs in the ring finger after the player misses a tackle and catches the digit on the shirt of the opposing player.  Surgical repair is required for zone I-type injuries.

 

REFERENCES: Moiemen NS, Elliot D: Primary flexor tendon repair in zone I.  J Hand Surg Br 2000;25:78-84.

Strickland JW: Flexor tendon injuries: I. Foundations of treatment.  J Am Acad Orthop Surg 1995;3:44-54.

 

94.       Storage of musculoskeletal allografts by cryopreservation is achieved by

 

1-         replacing water in the tissue with alcohol to a moisture level of 5% and then using a vacuum process to remove the alcohol from the tissue. 

2-         maintaining maximum cellular viability of fresh tissue without long-term storage.

3-         using chemicals to remove cellular water and controlled rate freezing to prevent ice crystal formation.

4-         freezing the graft twice and packaging the tissue without solution at minus
80 degrees C.

5-         freezing the graft in water without an antibiotic solution soak during quarantine, with final storage in liquid nitrogen.

 

PREFERRED RESPONSE: 3

 

DISCUSSION: Cryopreservation uses chemicals to remove cellular water and controlled rate freezing to prevent ice crystal formation.  The tissue is procured, cooled to wet ice temperature for quarantine, and then stored in a container with cryoprotectant solution of dimethyl

sulfoxide or glycerol which displaces the cellular water.  The controlled rate freezing is then done to prevent ice crystal formation.  Fresh allografts are not frozen in order to maintain maximum cellular viability, and this process limits the shelf life of osteochondral allografts.  Freeze-drying involves replacement of water in the tissue with alcohol to a moisture level of

5% and then uses a vacuum process to remove the alcohol from the tissue.  Preparation of fresh frozen grafts involves freezing the graft twice and packaging the tissue without solution at

minus 80 degrees C. 

 

REFERENCES: American Association of Tissue Banks: Standards for Tissue Banking.  MacLean, VA, American Association of Tissue Banks, 1999.

Vangsness CT Jr, Triffon MJ, Joyce MJ, et al: Soft tissue allograft reconstruction of the human knee: A survey of the American Association of Tissue Banks.  Am J Sports Med 1996;24:230-234.

Brautigan BE, Johnson DL, Caborn DM, et al: Allograft tissues, in DeLee JC, Drez D Jr (eds): Orthopaedic Sports Medicine: Principles and Practice.  Philadelphia, PA, WB Saunders, 2003, pp 205-213.

 

95.       A 15-year-old wrestler sustains an abduction, hyperextension, and external rotation injury to his right shoulder.  The MRI scan findings shown in Figures 27a and 27b are most consistent with

 

1-         an avulsion of the lesser tuberosity.

2-         a midsubstance tear of the capsule.

3-         a tear of the anterior inferior labrum.

4-         a tear of the subscapularis.

5-         a tear of the humeral insertion of the inferior glenohumeral ligament.

 

PREFERRED RESPONSE: 5

 

DISCUSSION: An isolated avulsion of the lesser tuberosity occurs very rarely and usually is found in 12- and 13-year-old adolescents.  The MRI scans reveal a tear of the humeral attachment of the inferior glenohumeral ligament, a so-called HAGL lesion.  This injury to the inferior glenohumeral ligament occurs much less commonly than the classic Bankart lesion (anterior inferior labral tear).  A tear of the subscapularis occurs with a similar mechanism of injury but generally occurs in older individuals. 

 

REFERENCES: Bokor DJ, Conboy VB, Olson C: Anterior instability of the glenohumeral joint with humeral avulsion of the glenohumeral ligament: A review of 41 cases.  J Bone Joint Surg Br 1999;81:93-96.

Wolf EM, Cheng JC, Dickson K: Humeral avulsion of the inferior glenohumeral ligaments as a cause of anterior shoulder instability.  Arthroscopy 1995;11:600-607.

 

96.       Closed-chain exercise differs from open-chain exercise in which of the following ways?

 

1-         Distal portion of the extremity is free during exercise

2-         More commonly used in upper extremity exercise

3-         Predictable movement is produced by co-contraction of muscles

4-         Joint compression is decreased

5-         Usually involves a single joint

 

PREFERRED RESPONSE: 3

 

DISCUSSION: Closed-chain exercise requires the distal portion of the extremity to be fixed.  It is more commonly used in lower extremity exercise, and movement is produced by co-contraction of muscles.  Joint compression is increased, and multiple joints are involved with closed-chain exercise.  In open-chain exercise, the distal portion of the extremity is free.

 

REFERENCES: Braddom RL (ed): Physical Medicine and Rehabilitation, ed 2.  Philadelphia, PA, Saunders, 2000, pp 975-976.

Childs DC, Irrang JJ: The language of exercise and rehabilitation, in Delee JC, Drez D (eds): Orthopaedic Sports Medicine, ed 2.  Philadelphia, PA, WB Saunders, 2003, vol 1, p 329.

 

97.       What procedure can eliminate a sulcus sign?

 

1-         Rotator interval closure

2-         SLAP repair

3-         Bankart repair

4-         Supraspinatus repair

5-         Subacromial decompression

 

PREFERRED RESPONSE: 1

 

DISCUSSION: A sulcus sign represents inferior subluxation of the shoulder.  The elimination of this sign and correction of the inferior subluxation is best achieved through either an open or arthroscopic rotator interval closure.  A SLAP repair stabilizes the biceps anchor but does not affect the sulcus sign.  A Bankart repair, which corrects anterior-inferior laxity, is not sufficient to eliminate a sulcus sign.  Subacromial decompression and supraspinatus repairs have no effect on inferior subluxation.

 

REFERENCES: Field LD, Warren RF, O’Brien SJ, et al: Isolated closure of rotator interval defects for shoulder instability.  Am J Sports Med 1995;23:557-563.

Cole BJ, Rodeo SA, O’Brien SJ, et al: The anatomy and histology of the rotator interval capsule of the shoulder.  Clin Orthop 2001;390:129-137.

 

98.       An eversion mechanism of injury is associated with which of the following ankle conditions?

 

1-         Peroneal tendon splits

2-         Osteochondral talar dome fracture

3-         Superficial peroneal nerve palsy

4-         Anterolateral soft-tissue impingement

5-         Fracture of the anterior colliculus

 

PREFERRED RESPONSE: 5

 

DISCUSSION: A fracture of the anterior colliculus is typically the result of an eversion mechanism resulting in a bony avulsion of the deltoid ligament from the anterior colliculus of the medial malleolus.  An inversion ankle injury typically involves ligamentous damage to the lateral ligaments of the ankle to include the anterior talofibular ligament and calcaneofibular ligament.  Acute and particularly chronic ankle sprains also can have associated injuries.  The inversion mechanism has been implicated in osteochondral and transchondral talar dome lesions, producing splits in the peroneus tendons, and in the development of meniscoid and soft-tissue impingement lesions in the anterolateral ankle.  An inversion mechanism can also stretch the superficial peroneal nerve, leading to pain and paresthesias along its distribution. 

 

REFERENCES: Casillas MM: Ligament injuries of the foot and ankle in the athlete, in DeLee JC, Drez D, Miller MD (eds): Orthopaedic Sports Medicine: Principles and Practice, ed 2.  Philadelphia, PA, WB Saunders, 2003, pp 2323-2357.

Garrick JG (ed): Orthopaedic Knowledge Update: Sports Medicine 3.  Rosemont, IL, American Academy of Orthopaedic Surgeons, 2004, pp 233-245.

 

99.       Which of the following anatomic structures are in contact with internal impingement in the throwing athlete?

 

1-         Humerus and posterior-superior glenoid

2-         Humerus and anterior inferior glenoid

3-         Humerus and acromion

4-         Biceps and acromion

5-         Rotator cuff and acromion

 

PREFERRED RESPONSE: 1

 

DISCUSSION: Internal impingement occurs in the late cocking phase of throwing with humeral head abduction and maximal external rotation.  It is a physiologic phenomenon occurring in

85% of patients undergoing arthroscopy for various indications in one study.  Internal impingement is defined as impingement of the posterior-superior rotator cuff between the humerus and posterior-superior glenoid rim.  Symptomatic internal impingement is felt to be due to the frequency and magnitude of the impingement in throwers.

 

REFERENCES: Koval KJ (ed): Orthopaedic Knowledge Update 7.  Rosemont, IL, American Academy of Orthopaedic Surgeons, 2002, p 252.

Paley KJ, Jobe FW, Pink MM, et al: Arthroscopic findings in the overhand throwing athlete: Evidence for posterior internal impingement of the rotator cuff.  Arthroscopy 2000;16:35-40.

 

100.     Second impact syndrome (SIS) after head injury is characterized by which of
the following?

 

1-         Gradual progression of neurologic symptoms

2-         Preventable by restricting return to play until symptom-free

3-         Excellent prognosis for full recovery

4-         Rarely involves brain stem compromise

5-         CT rarely shows brain edema

 

PREFERRED RESPONSE: 2

 

DISCUSSION: SIS is a devastating but preventable complication of head injury.  It occurs when return to activities is allowed prior to complete resolution of the symptoms of the first head injury.  A second, sometimes trivial, head injury can lead to a devastating series of events that can result in sudden death.  The symptoms tend to progress rapidly and often involve the brain stem.  The prognosis is poor.

 

REFERENCES: Cantu RC: Second-impact syndrome.  Clin Sports Med 1998;17:37-44.

Saunders RL, Harbaugh RE: Second impact in catastrophic contact-sports head trauma.  JAMA 1984;252:538-539.

Stevenson KL, Adelson PD: Pediatric sports-related head injuries, in Delee JC, Drez D (eds): Orthopaedic Sports Medicine: Principles and Practice, ed 2.  Philadelphia, PA, WB Saunders, 2003, vol 1, p 781.